Sie sind auf Seite 1von 223

PART 1 A A.

More than 1 year but


MICROECONOMICS less than 5 years.
193 QUESTIONS
B. Five or more years.
[1] Source: CMA 1285 1-
15 C. A period of time long
In the short run, a purely enough to turn over top
competitive firm management.
operating at a loss will
D. A period of time long
A. Shut down. enough for the firm to
make all resource costs
B. Continue to operate variable in nature.
as long as price exceeds
average variable costs.
[3] Source: CMA 1285 1-
C. Raise the price of its 17
product. The marginal utility of a
good refers to
D. Reduce the size of
its plant to lower fixed A. The point at which
costs. the consumer's total
utility is
maximized.
[2] Source: CMA 1285 1-
16 B. The point at which
In an analysis of the consumer's total
economic costs of utility is
Production, long run minimized.
means
C. The change in the
amount of the good [5] Source: CMA 1286 1-7
consumed If a rent control law in a
that increases total competitive housing
utility by one unit. market
establishes a maximum
D. The change in total or ceiling rent that is
utility when consumption above the
of the market or equilibrium
good increases by one rent,
unit.
A. The law has no
effect on the rental
[4] Source: CMA 1286 1-4 market.
Economies and
diseconomies of scale B. A surplus of rental
are important housing units will result.
determinants of the
C. Supply will decrease
A. Type of product as price increases.
demand faced by
individual firms. D. Demand will increase
as price increases.
B. Market demand
curve.
[6] Source: CMA 0687 1-1
C. Pattern of costs in The distribution of
the long run. income among
households in the United
D. Law of diminishing States is primarily
returns. determined by
A. Transfer payments to
households.
[8] Source: CMA 1287 1-2
B. Household If the demand for
purchases of goods and cigarettes in New York is
services. relatively
elastic, and New York
C. The ownership of imposes high taxes on
factors of production. cigarettes
that result in higher
D. Government taxes. cigarette prices, then in
New York

[7] Source: CMA 0687 1- A. The quantity of


12 cigarettes demanded
Local electric utilities are would
considered to be natural increase.
monopolies because for
these firms B. The demand for
cigarettes would
A. Supply curves are increase.
upward sloping.
C. The demand curve
B. Demand curves are for cigarettes would
upward sloping. become
vertical.
C. Average total costs
never fall. D. Expenditures on
cigarettes would fall.
D. Significant
economies of scale are
present. [9] Source: CMA 1287 1-5
The quantity of output a B. In which both fixed
competitive firm can and variable inputs are
supply at any price is employed.
determined in part by
C. That is subject to
A. Average household economies of scale.
income.
D. That always
B. Consumer tastes and produces economic
preferences. profits.

C. Input prices.
[11] Source: CMA 1287 1-
D. The distribution of 12
income among Natural monopoly
households. conditions, which often
lead to
economic regulation,
[10] Source: CMA 1287 1- refer to
10
In the economic theory of A. Rising marginal
production and cost, the costs.
short
run is defined to be a B. Elastic consumer
production process demand for the product.

A. Which spans a time C. Declining average


period of less than one costs.
year in
length. D. Consumer demand
for the product that is
strongly
influenced by the A. Perfectly inelastic.
business cycle.
B. Elastic.

[Fact Pattern #1] C. Inelastic.


The theory of price
elasticity of consumer D. Unit elastic.
demand is helpful when a
firm is determining price
changes for a consumer [13] Source: CMA 1288 1-
good or service. The 23
formula for the (Refers to Fact Pattern
coefficient of elasticity, #1)
E, is If the coefficient of
elasticity is zero, then
(relative change in the consumer
quantity demanded) demand for the product is
E= said to be
--------------------------------------
(relative change A. Perfectly inelastic.
in price)
B. Elastic.
[12] Source: CMA 1288 1-
22 C. Inelastic.
(Refers to Fact Pattern
#1) D. Unit elastic.
If the coefficient of
elasticity is two, then the
consumer [14] Source: CMA 1288 1-
demand for the product is 27
said to be
All of the following are the
true about perfect
competition A. Supply curve will
except that shift to the left.

A. There is free market B. Quantity demanded


entry without large will move farther down
capital the
costs for entry. demand curve.

B. There are many firms C. Demand will shift to


participating in the the left.
market.
D. Demand curve will
C. In the long run, an shift to the right.
increase in profit will
have no
effect on the number of [16] Source: CMA 1288 1-
firms in the market. 29
All of the following are
D. Firms are price characteristics of
takers. monopolistic
competition except that

[15] Source: CMA 1288 1- A. The firms sell a


25 homogeneous product.
If the average household
income increases and B. The firms tend not to
there is recognize the reaction of
relatively little change in competitors when
the price of a normal determining prices.
good, then
C. Individual firms have [18] Source: CMA 0689 1-
some control over the 20
price The measurement that
of the product. uses the factors of
production as
D. The consumer inputs in physical terms
demand curve is highly is
elastic.
A. Economic efficiency.

[17] Source: CMA 1288 1- B. Opportunity cost.


30
The cyclical and C. Technological
seasonal fluctuations in efficiency.
consumer demand for the
products produced by an D. Comparative
oligopoly are advantage.
characteristic of

A. Differentiated [19] Source: CMA 0689 1-


products. 21
The measurement of
B. Sticky prices. using resources now in
lieu of
C. Kinked demand alternative uses of the
curves. resources is

D. Homogeneous A. Economic efficiency.


products.
B. Opportunity cost.
C. Comparative [21] Source: CMA 0689 1-
advantage. 24
When making a decision
D. Absolute advantage. to increase the robotic
automation equipment in
an existing facility, a firm
[20] Source: CMA 0689 1- takes all of the following
22 into consideration except
The principle of
substitution states that A. Economies of scale.

A. The costs of two B. Opportunity cost.


factors of production are
equal. C. Technological
efficiency.
B. Profit maximization
is maintained while D. The initial cost of the
choosing current facility.
either factor of
production.
[22] Source: CMA 0689 1-
C. The firm chooses 26
more of the less A decrease in the price of
expensive a complementary good
factor of production. will

D. The firm chooses A. Shift the demand


more of the more curve of the joint
expensive commodity to
factor of production. the left.
B. Increase the price
paid for a substitute C. The demand curve
good. will shift to the right.

C. Shift the supply D. The demand curve


curve of the joint will shift to the left.
commodity to
the left.
[24] Source: CMA 1289 1-
D. Shift the demand 4
curve of the joint If the elasticity of
commodity to demand for a normal
the right. good is estimated to be
1.5, a 10% reduction in
its price would cause
[23] Source: CMA 0689 1-
29 A. Total revenue to fall
If a product is part of the by 10%.
consumers' basket of
goods, and the Consumer B. Total revenue to fall
Price Index increased 7% by 15%.
for the year while the
price of this normal good C. Quantity demanded
increased 3%, then to rise by 15%.

A. The supply curve will D. Demand to decrease


shift to the left. by 10%.

B. Neither the demand


curve nor the supply [25] Source: CMA 1289 1-
curve 11
will be affected.
An industry that is B. Is perfectly elastic.
oligopolistic would be
best characterized by C. Responds as an
inferior good.
A. One firm selling a
product with no close D. Is perfectly inelastic.
substitutes.

B. The absence of the [27] Source: CMA 0690 1-


profit-maximizing goal. 20
If oil producers and
C. Significant barriers retailers were to increase
to entry. the price of
gasoline for cars during
D. Horizontal or flat the summer season by
demand curves for the $.05 per
output of gallon, these suppliers
individual firms. anticipate that the
demand for
gasoline
[26] Source: CMA 0690 1-
19 A. Is relatively elastic.
If a normal good
competes with three B. Is relatively
similar goods, and all four inelastic.
goods give the consumer
equal utils of C. Responds as an
satisfaction, the demand inferior good.
for the normal good
D. Is perfectly inelastic.
A. Is relatively elastic.
[28] Source: CMA 0690 1- establishes a
21 comfortable life-style
The Waymand family (i.e., electricity, gas for
typically ate hamburger home heating), based on
as a staple in their diet. the assumption that the
In the last few years, the demand
family's income has
doubled, and they have A. Is relatively elastic.
now replaced hamburger
with B. Is perfectly elastic.
steak as a staple in their
diet. This is an example C. Is relatively
in which inelastic.
the demand for
hamburger D. Is perfectly inelastic.

A. Is relatively elastic.
[30] Source: CMA 0690 1-
B. Is perfectly elastic. 23
The demand curve for a
C. Is relatively normal good is
inelastic.
A. Upward sloping
D. Responds as an because firms produce
inferior good. more at
higher prices.

[29] Source: CMA 0690 1- B. Upward sloping


22 because higher-priced
Utility companies can goods are
ordinarily price their of higher quality.
product, a good that
C. Vertical. Which one of the
following examples best
D. Downward sloping depicts the law
because of the income of diminishing returns?
and
substitution effects of A. Small electric
price changes. generating plants are
less efficient
than large plants.
[31] Source: CMA 0690 1-
24 B. A manufacturing
Which one of the company purchases its
following changes will supplier
cause the demand of materials.
curve for gasoline to shift
to the left? C. An automobile
assembly plant has lower
A. The price of gasoline per-unit
increases. costs at 85% of
capacity than at 95% of
B. The supply of capacity.
gasoline decreases.
D. Passenger airplanes
C. The price of cars operate at high costs per
increases. passenger when they
fly half-empty.
D. The price of cars
decreases.
[33] Source: CMA 0690 1-
27
[32] Source: CMA 0690 1-
26
A corporation's net The marginal cost curve
income as presented on is the supply curve for
its income the firm in
statement is usually
A. Pure monopoly.
A. More than its
economic profits because B. Monopolistic
opportunity costs are competition.
not considered in
calculating net C. Pure competition.
income.
D. Oligopoly.
B. More than its
economic profits because
economists do not [35] Source: CMA 0690 1-
consider interest 30
payments to be An improvement in
costs. technology that in turn
leads to
C. Equal to its improved worker
economic profits. productivity would most
likely result in
D. Less than its
economic profits because A. A shift to the right in
accountants include the supply curve and a
labor costs, while lowering of the price of
economists the output.
exclude labor costs.
B. A shift to the left in
the supply curve and a
[34] Source: CMA 0690 1- lowering of the price of
29 the output.
[37] Source: CMA 1290 1-
C. An increase in the 2
price of the output if Tennis rackets and
demand is tennis balls are
unchanged.
A. Substitute goods.
D. Wage increases.
B. Independent goods.

[36] Source: CMA 1290 1- C. Inferior goods.


1
The existence of D. Complementary
economic profit in pure goods.
monopoly will

A. Have no influence on [38] Source: CMA 1290 1-


the number of firms in the 3
industry. Which one of the
following is a
B. Lead to a decline in characteristic of pure
the number of firms in the competition?
industry.
A. Mutual
C. Lead to an increase interdependence.
in product supply.
B. Significant research
D. Lead to a decline in and development
market prices for programs.
substitutes.
C. Product
differentiation.
D. Standardized
product. [40] Source: CMA 1290 1-
5
A government price
[39] Source: CMA 1290 1- support program will
4
Which one of the A. Lead to surpluses.
following statements
about supply and B. Lead to shortages.
demand is true?
C. Improve the
A. If supply increases rationing function of
and demand remains prices.
constant,
equilibrium price will D. Encourage firms to
rise. leave the industry.

B. If demand increases
and supply decreases, [41] Source: CMA 1290 1-
equilibrium price will 6
fall. Price ceilings

C. If demand increases A. Are illustrated by


and supply increases, government price support
equilibrium quantity will programs in agriculture.
fall.
B. Create prices greater
D. If demand increases than equilibrium prices.
and supply decreases,
equilibrium price will C. Create prices below
increase. equilibrium prices.
D. Result in persistent B. Is strongly
surpluses. influenced by diminishing
returns.

[42] Source: CMA 1290 1- C. Is the same as the


7 rising portion of the
Entry into monopolistic marginal
competition is cost curve.

A. Blocked. D. Is ordinarily U-
shaped.
B. Difficult, with
significant obstacles.
[44] Source: CMA 1290 1-
C. Rare, as significant 10
capital is required. A normal profit is

D. Relatively easy, with A. The same as an


only a few obstacles. economic profit.

B. The same as the


[43] Source: CMA 1290 1- accountant's bottom line.
8
The long-run average C. An explicit or out-of-
total cost curve pocket cost.

A. Is horizontal and D. A cost of resources


parallel to the demand from an economic
curve. perspective.
[45] Source: CMA 1290 1-
9 C. Identified with an
Which one of the industry in which
following is not a factor economies of
contributing to scale are rapidly
economies of scale? exhausted.

A. Labor specialization. D. Identified with an


industry in which
B. Use of by-products. economies of
scale are few and
C. Efficient use of diseconomies are quickly
capital equipment. incurred.

D. Diminishing returns.
[47] Source: CMA 1290 1-
12
[46] Source: CMA 1290 1- A high concentration
11 ratio is
A natural monopoly is
A. An indicator of
A. Identified with a one- monopolistic power.
firm industry with
significant B. An indicator of a
economies of scale and highly competitive
in which unit costs are industry.
minimized.
C. Consistent with the
B. An important part of law of demand.
the analysis of
monopolistic
competition.
D. Consistent with C. Steel industry.
monopolistic
competition. D. Auto industry.

[48] Source: CMA 1290 1- [50] Source: CMA 1290 1-


13 15
A supply curve illustrates Marginal revenue is
the relationship between
A. Equal to price in
A. Price and quantity monopolistic
supplied. competition.

B. Price and consumer B. The change in total


tastes. revenue associated with
increasing prices.
C. Price and quantity
demanded. C. Greater than price in
pure competition.
D. Supply and demand.
D. The change in total
revenue associated with
[49] Source: CMA 1290 1- producing and selling
14 one more unit.
A good example of
monopolistic competition
is the [51] Source: CMA 1290 1-
16
A. Agriculture market. The kinked demand curve
is associated with
B. Fast food industry.
A. The analysis of D. Elasticity of demand
agricultural markets. for apples is less than
1.0.
B. The analysis of
monopolistic
competition. [53] Source: CMA 1290 1-
18
C. The analysis of pure The firm's short-run
competition. supply curve is derived
from the
D. The analysis of
oligopoly. A. Average total cost
curve.

[52] Source: CMA 1290 1- B. Fixed cost curve.


17
If the price of apples C. Marginal cost curve.
declines and total
revenue received by D. Total cost curve.
the firm increases, the

A. Demand for apples is [54] Source: CMA 0691 1-


elastic. 13
If both the supply and the
B. Demand for apples is demand for a good
inelastic. increase, the
market price will
C. Elasticity of demand
for apples is 1.0. A. Rise only in the case
of an inelastic supply
function.
B. Fall only in the case
of an inelastic supply
function. [56] Source: CMA 0691 1-
15
C. Not be predictable Economic rent is the
with only these facts. total price paid for land
and other
D. Rise only in the case natural resources when
of an inelastic demand there is a(n)
function.
A. Completely elastic
supply function.
[55] Source: CMA 0691 1-
14 B. Completely fixed
Economic goods are total supply.
considered scarce
resources because C. Fixed demand
they schedule.

A. Cannot be increased D. Artificial market


in quantity. price.

B. Are not produced in


adequate quantities. [57] Source: CMA 0691 1-
19
C. Do not exist in When the federal
adequate quantities to government imposes
satisfy all health and safety
demands for them. regulations on certain
products, one of the most
D. Are limited to man- likely
made goods. results is
A. Greater consumption [59] Source: CMA 0692 1-
of the product. 29
The change in total
B. Lower prices for the product resulting from
product. the use of one
unit more of the variable
C. Greater tax revenues factor is known as
for the federal
government. A. The point of
diminishing average
D. Higher prices for the productivity.
product.
B. Marginal product.

[58] Source: CMA 0692 1- C. Marginal cost.


28
The sum of the average D. The point of
fixed costs and the diminishing marginal
average productivity.
variable costs for a given
output is known as
[60] Source: CMA 0692 1-
A. Long-run average 30
cost. When long-run average
cost is declining over a
B. Average product. range of
increasing output, the
C. Total cost. firm is experiencing

D. Average total cost. A. Increasing fixed


costs.
[62] Source: CMA 1292 1-
B. Technological 2
efficiency. A perfectly inelastic
supply curve in a
C. Decreasing returns. competitive market

D. Economies of scale. A. Implies a vertical


demand curve.

[61] Source: CMA 1292 1- B. Exists when firms


1 cannot vary input usage.
In any competitive
market, an equal C. Implies a horizontal
increase in both market supply curve.
demand and supply can
be expected to always D. Can exist only in the
long run.
A. Increase both price
and market-clearing
quantity. [63] Source: CMA 1292 1-
3
B. Decrease both price Government price
and market-clearing regulations in
quantity. competitive markets that
set maximum or ceiling
C. Increase market- prices below the
clearing quantity. equilibrium price
will in the short run
D. Increase price.
A. Cause demand to
decrease.
B. Cause supply to [65] Source: CMA 1292 1-
increase. 5
In the theory of demand,
C. Create shortages of the marginal utility per
that product. dollar of a
product
D. Produce a surplus of
the product. A. Increases when
consumption expands.

[64] Source: CMA 1292 1- B. Decreases when


4 consumption expands.
In competitive product
markets, equilibrium C. Explains why short-
price in the run supply curves are
long run is upward
sloping.
A. A fair price all
consumers can afford. D. Increases as more
units of a variable input
B. Set equal to the total are
costs of production. added to the production
process.
C. Set equal to the total
fixed costs of production.
[66] Source: CMA 1292 1-
D. Set equal to the 6
marginal costs of In the short run, the
production. supply curve in a
competitive market
shows a positive A. Each additional unit
relationship between of labor is less efficient
price and quantity than
supplied because the previous unit.

A. Consumers will only B. As more labor is


buy more of a product at added to a factory,
a increases in
higher price. output will diminish in
the short run.
B. Of the law of
diminishing returns. C. Increasing the size
of a factory will result in
C. As the size of a lower
business firm increases, average costs.
price must
rise. D. Increasing the size
of a factory will result in
D. Increases in output lower
imply a shift in consumer total costs.
preferences, allowing a
higher price.
[68] Source: CMA 1292 1-
8
[67] Source: CMA 1292 1- When compared with
7 firms in perfectly
Because of the existence competitive
of economies of scale, markets, monopolists
business ordinarily
firms may find that
A. Use more advertising
to increase sales.
C. Advertising.
B. Charge a higher
price and produce a D. Heterogeneous
higher rate of products.
output.

C. Use more capital and [70] Source: CMA 1292 1-


less labor to avoid 10
problems When markets are
with workers. perfectly competitive,
consumers
D. Charge a higher
price and produce a A. Are able to avoid the
lower rate of problem of diminishing
output. returns.

B. Have goods and


[69] Source: CMA 1292 1- services produced at the
9 lowest
Economic markets that cost in the long run.
are characterized by
monopolistic C. Do not receive any
competition have all of consumer surplus unless
the following producers choose to
characteristics except overproduce.

A. One seller of the D. Must search for the


product. lowest price for the
products
B. Economies or they buy.
diseconomies of scale.
[71] Source: CMA 1292 1- A. All the dollar costs
11 employers pay for all
The manner in which inputs
cartels set and maintain purchased.
price above
the competitive market B. The opportunity cost
price is to of all inputs minus the
dollar
A. Avoid product cost of those inputs.
differentiation in order to
decrease C. The difference
demand for the between all implicit and
product. explicit
costs of the business
B. Advertise more so firm.
market demand
increases. D. The sum of all
explicit and implicit costs
C. Increase costs so of the
price must rise. business firm.

D. Require cartel
members to restrict [73] Source: CMA 1292 1-
output. 13
In a competitive market
for labor in which
[72] Source: CMA 1292 1- demand is
12 stable, if workers try to
The definition of increase their wage,
economic cost is
A. Employment must
fall.
B. Government must set D. Mutual
a maximum wage below interdependence of firm
the pricing and output
equilibrium wage. decisions.

C. Firms in the industry


must become smaller. [75] Source: CMA 1292 1-
15
D. Product supply must In microeconomics, the
decrease. distinguishing
characteristic of the
long run on the supply
[74] Source: CMA 1292 1- side is that
14
The distinguishing A. Only supply factors
characteristic of determine price and
oligopolistic markets is output.

A. A single seller of a B. Only demand factors


homogeneous product determine price and
with no output.
close substitute.
C. Firms are not
B. A single seller of a allowed to enter or exit
heterogeneous product the industry.
with no
close substitute. D. All inputs are
variable.
C. Lack of entry and
exit barriers in the
industry.
[76] Source: CMA 1292 1- B. Minimum price below
17 the equilibrium price.
Any business firm that
has the ability to control C. Maximum price
the price of below the equilibrium
the product it sells price.

A. Faces a downward- D. Minimum price above


sloping demand curve. the equilibrium price.

B. Has a supply curve


that is horizontal. [78] Source: CMA 1293 1-
3
C. Has a demand curve A natural monopoly
that is horizontal. exists because

D. Will sell all output A. The firm owns


produced. natural resources.

B. The firm holds


[77] Source: CMA 1292 1- patents.
18
The competitive model of C. Economic and
supply and demand technical conditions
predicts that permit only
a surplus can arise only if one efficient supplier.
there is a
D. The government is
A. Maximum price the only supplier.
above the equilibrium
price.
[79] Source: CMA 1293 1- monopoly and
10 monopolistic
If a group of consumers competition, firms
decide to boycott a produce at
particular an output at which
product, the expected
result would be A. Price equals
marginal cost.
A. An increase in the
product price to make up B. Average costs are
lost minimized.
revenue.
C. Price equals average
B. A decrease in the cost.
demand for the product.
D. Marginal cost equals
C. An increase in marginal revenue.
product supply because
of
increased availability. [81] Source: CMA 1293 1-
29
D. That demand for the Price tends to fall in
product would become competitive markets
completely inelastic. when there is a(n)

A. Increase in demand
[80] Source: CMA 1293 1- for the product.
28
In markets that are B. Decrease in quantity
imperfectly competitive, demanded of the product.
such as
C. Decline in available supply will
labor.
A. Increase the
D. Increase in interest equilibrium price and the
rates. equilibrium
quantity exchanged.

[82] Source: CMA 1293 1- B. Decrease the


30 equilibrium price and the
Product demand equilibrium
becomes more elastic quantity exchanged.
the
C. Increase the
A. Greater the number equilibrium price and
of substitute products decrease the
available. equilibrium quantity
exchanged.
B. Greater the
consumer income. D. Decrease the
equilibrium price and
C. Greater the elasticity increase the
of supply. equilibrium quantity
exchanged.
D. Higher the input
costs.
[84] Source: CMA 1288 1-
28
[83] Source: CMA 1288 1- A characteristic of a
26 monopoly is that
In relation to the laws of
supply and demand, an
increase in
A. A monopoly will fixed costs of $10,000
produce when marginal and total variable costs
revenue of $15,000,
is equal to marginal the average total cost
cost. per unit is

B. There is a unique A. $20.


relationship between the
market B. $30.
price and the quantity
supplied. C. $50.

C. In optimizing profits, D. $25.


a monopoly will increase
its
supply curve to where [86] Source: CMA 1289 1-
the demand curve 8
becomes In the short run in perfect
inelastic. competition, a firm
maximizes
D. There are multiple profit by producing the
prices for the product to rate of output at which
the the price is
consumer. equal to

A. Total cost.
[85] Source: CMA 1289 1-
7 B. Total variable cost.
If a firm currently
producing 500 units of C. Average fixed costs.
output incurs total
D. Marginal cost.
A. All possible
combinations of two
[87] Source: CMA 1289 1- different product
9 quantities that a
Because of economies of producer would be willing
scale, as output from to sell.
production
expands, B. All possible
combinations of two
A. The short-run different product
average cost of quantities that will
production yield the same level of
decreases. satisfaction
to the consumer.
B. The long-run average
cost of production C. Combinations of two
increases. different product
quantities
C. The long-run total that are possible, given
cost decreases. a consumer's income and
the
D. The slope of the prices of the two
demand curve increases. products.

D. A consumer's
[88] Source: CMA 0694 1- indifference between
2 varying levels
An indifference curve of income and price
represents changes.
[89] Source: CMA 0694 1- D. 3.80
3
As the price for a
particular product [90] Source: CMA 0694 1-
changes, the quantity 6
of the product demanded If a product's demand is
changes according to the elastic and there is a
following schedule. decrease in
price, the effect will be
Total Quantity
Price A. A decrease in total
Demanded revenue.
per Unit
-------------- -------- B. No change in total
100 $50 revenue.
150 45
200 40 C. A decrease in total
225 35 revenue and the demand
230 30 curve
232 25 shifts to the left.
The price elasticity of
demand for this product D. An increase in total
when the revenue.
price decreases from $50
to $45 is
[91] Source: CMA 0694 1-
A. 0.20 13
The movement along the
B. 10.00 demand curve from one
price-quantity
C. 0.10 combination to another is
called a(n)
[93] Source: CMA 1294 1-
A. Change in demand. 6
An oligopolist faces a
B. Shift in the demand "kinked" demand curve.
curve. This
terminology indicates
C. Change in the that
quantity demanded.
A. When an oligopolist
D. Increase in demand. lowers its price, the other
firms in the oligopoly
will match the price
[92] Source: CMA 0694 1- reduction,
14 but if the oligopolist
All of the following are raises its price, the other
complementary goods firms
except will ignore the price
change.
A. Margarine and
butter. B. An oligopolist faces
a non-linear demand for
B. Cameras and rolls of its
film. product, and price
changes will have little
C. VCRs and video effect on
cassettes. demand for that
product.
D. Razors and razor
blades. C. An oligopolist can
sell its product at any
price, but
after the "saturation
point," another C. Elastic.
oligopolist will
lower its price and, D. Inelastic.
therefore, shift the
demand curve
to the left. [95] Source: CMA 1294 1-
8
D. Consumers have no Monopolistic competition
effect on the demand is characterized by
curve,
and an oligopolist can A. A relatively large
shape the curve to number of sellers who
optimize its produce
own efficiency. differentiated products.

B. A relatively small
[94] Source: CMA 1294 1- number of sellers who
7 produce
In the pharmaceutical differentiated products.
industry where a diabetic
must have C. A monopolistic
insulin no matter what market where the
the cost and where there consumer is
is no other persuaded that there is
substitute, the diabetic's perfect competition.
demand curve is best
described as D. A relatively large
number of sellers who
A. Perfectly elastic. produce a
standardized product.
B. Perfectly inelastic.
C. The population in the
[96] Source: CMA 1294 1- market area is large.
19
If a product has a price D. Few good
elasticity of demand of substitutes are available
2.0, the for the
demand is said to be product.

A. Perfectly elastic.
[98] Source: CMA 0695 1-
B. Perfectly inelastic. 16
Which one of the
C. Elastic. following statements is
not true of
D. Inelastic. indifference curves?

A. Indifference curves
[97] Source: CMA 0695 1- slope downward to the
15 right,
Demand for a product indicating goods are
tends to be price substitutable.
inelastic if
B. Indifference curves
A. The product is indicate that more goods
considered a luxury item. are
preferable to fewer
B. Few good goods.
complements for the
product are C. The marginal utility
available. of a good decreases as
consumption of the
good increases.
[100] Source: CMA 0695
D. Indifference curves 1-18
cross at their equilibrium A market with many
point. independent firms, low
barriers to
entry, and product
[99] Source: CMA 0695 1- differentiation is best
17 classified as
Which one of the
following is not a key A. A monopoly.
assumption of
perfect competition? B. Monopolistic
competition.
A. Firms sell a
homogeneous product. C. An oligopoly.

B. Customers are D. Pure competition.


indifferent about which
firm they
buy from. [101] Source: CMA 0695
1-19
C. The level of a firm's Which one of the
output is small relative to following would cause
the the demand curve
industry's total output. for a normal good to shift
to the left?
D. Each firm can price
its product above the A. A rise in the price of
industry a substitute product.
price.
B. A rise in average
household income.
[Fact Pattern #2]
C. A rise in the price of
a complementary Number of Total
commodity. Average
Workers Product Units
D. A change in Selling Price
consumers' tastes in --------- ------------- -------------
favor of the 10 20
commodity. $50.00
11 25
49.00
[102] Source: CMA 0695 12 28
1-20 47.50
An increase in the market
supply of beef would [103] Source: CMA 1295
result in 1-17
a(n) (Refers to Fact Pattern
#2)
A. Increase in the price The marginal physical
of beef. product when one worker
is added
B. Decrease in the to a team of 10 workers
demand for beef. is

C. Increase in the price A. 1 unit.


of pork.
B. 8 units.
D. Increase in the
quantity of beef C. 5 units.
demanded.
D. 25 units.
B. $225.00
[104] Source: CMA 1295
1-18 C. $105.00
(Refers to Fact Pattern
#2) D. $47.50
The marginal revenue per
unit when one worker is
added [106] Source: CMA 1285
to a team of 11 workers 1-18
is In preparing a cost-
volume-profit analysis for
A. $35.00 his candle
manufacturing business,
B. $225.00 Joe Stark is considering
raising his
C. $105.00 prices $1.00 per candle.
Stark is worried about
D. $47.50 the impact
the increase will have on
his volume of sales at
[105] Source: CMA 1295 craft fairs.
1-19 Stark is concerned about
(Refers to Fact Pattern the
#2)
The marginal revenue A. Elasticity of demand.
product when one worker
is added B. Substitution effect.
to a team of 11 workers
is C. Nature of supply.

A. $42.00 D. Maximization of
utility.
[108] Source: CMA 0696
1-4
[107] Source: CMA 0696 The law of diminishing
1-3 marginal utility states
Which one of the that
following statements
concerning pure A. Marginal utility will
monopolies is correct? decline as a consumer
acquires
A. The demand curve of additional units of a
a monopolist is perfectly specific product.
elastic.
B. Total utility will
B. The price at which a decline as a consumer
monopolist maximizes its acquires
profit is where price additional units of a
equals both marginal specific product.
cost and
marginal revenue. C. Declining utils cause
the demand curve to
C. A monopolist's slope
marginal revenue curve upward.
lies below
its demand curve. D. Consumers' wants
diminish with the
D. For a monopolist, passage of
there is a unique time.
relationship
between the price and
the quantity supplied. [109] Source: CMA 0696
1-5
If a product has a price [111] Source: CMA 0696
elasticity of demand of 1-7
2.0, the Monopolistic competition
demand is considered to is characterized by
be
A. A relatively large
A. Perfectly elastic. group of sellers who
produce
B. Perfectly inelastic. differentiated products.

C. Relatively elastic. B. A relatively small


group of sellers who
D. Relatively inelastic. produce
differentiated products.

[110] Source: CMA 0696 C. A monopolistic


1-6 market where the
If a diabetic must have consumer is
insulin no matter what persuaded that there is
the cost, the perfect competition.
diabetic's demand is
considered to be D. A relatively large
group of sellers who
A. Perfectly elastic. produce a
homogeneous product.
B. Perfectly inelastic.

C. Relatively inelastic. [112] Source: CMA 1296


1-1
D. Indifferent. If the federal government
regulates a product or
service in a
competitive market by price.
setting a maximum price
below the C. Produce the same
equilibrium price, what is output and charge a
the long-run effect? higher
price.
A. A surplus.
D. Produce
B. A shortage. substantially less and
charge a lower
C. A decrease in price.
demand.

D. No effect on the [114] Source: CMA 1296


market. 1-3
Which one of the
following statements
[113] Source: CMA 1296 concerning pure
1-2 monopolies is correct?
Compared with firms in a
perfectly competitive A. The demand curve of
market, a a monopolist is perfectly
monopolist tends to elastic.

A. Produce B. The point of profit


substantially less but maximization for a
charge a higher monopoly is
price. where average total
revenue equals average
B. Produce total
substantially more and cost.
charge a higher
C. The monopolist's Natural monopoly
marginal revenue curve conditions, which often
lies lead to
below the monopolist's governmental regulation,
demand curve. exist when

D. The supply curve of a A. Consumer demand


monopoly is perfectly for a product is perfectly
inelastic. elastic.

B. Marginal costs are


[115] Source: CMA 1296 rising.
1-4
In the long run, a firm C. Consumer demand is
may experience inversely related to the
increasing returns business cycle.
due to
D. Total average costs
A. Law of diminishing are declining.
returns.

B. Opportunity costs. [117] Source: CMA 1296


1-27
C. Comparative A horizontal merger is
advantage. best defined as a merger
of
D. Economies of scale.
A. Two or more firms in
the same industry.
[116] Source: CMA 1296
1-19 B. A manufacturing
company and its supplier.
Nevertheless, Parker
C. Two or more firms in believes that if she is
different industries. able to obtain
capital resources, she
D. Two or more firms in can gain market share
different stages of the from the two
production process. major competitors.

[118] Source: CMA 0697


[Fact Pattern #3] 1-1
Karen Parker wants to (Refers to Fact Pattern
establish an #3)
environmental testing The large capital outlay
company that would necessary for the
specialize in evaluating equipment is an
the quality of example of a(n)
water found in rivers and
streams. However, A. Entry barrier.
Parker has
discovered that she B. Minimum efficient
needs either certification scale.
or approval
from five separate local C. Created barrier.
and state governmental
agencies D. Production
before she can possibility boundary.
commence business.
Also, the necessary
equipment to begin would [119] Source: CMA 0697
cost several million 1-2
dollars. (Refers to Fact Pattern
#3)
The market structure D. Public goods.
Karen Parker is
attempting to enter is
best described as [Fact Pattern #4]

A. A natural monopoly. Total Units Average


Average Average
B. A cartel. of Product Fixed Cost
Variable Cost Total
C. An oligopoly. Cost
----------- ---------- -------------
D. Monopolistic -----------
competition. 6 $15.00
$25.00 $40.00
7 12.86
[120] Source: CMA 0697 24.00 36.86
1-3 8 11.25
The local video store's 23.50 34.75
business increased by 9 10.00
12% after the 23.75 33.75
movie theater raised its
prices from $6.50 to [121] Source: CMA 0697
$7.00. This is 1-4
an example of (Refers to Fact Pattern
#4)
A. Substitute goods. The total cost of
producing seven units is
B. Superior goods.
A. $90.02
C. Complementary
goods. B. $168.00
C. $258.02
Games Games
D. $280.00
Played Played
Previous New
[122] Source: CMA 0697 at Previous at New
1-5 Rate Rate
(Refers to Fact Pattern Rate Rate
#4) -------- ----
The marginal cost of ----------- --------
producing the ninth unit Regular
is weekday $10
$11 80 70
A. $23.50 Senior citizen 6 8
150 82
B. $23.75 Weekend 15 20
221 223
C. $25.75 Which one of the
following is correct?
D. $33.75
A. The regular weekday
and weekend demand is
[123] Source: CMA 0697 inelastic.
1-6
Long Lake Golf Course B. The regular weekday
has raised greens fees and weekend demand is
for a elastic.
nine-hole game due to an
increase in demand. C. The senior citizen
demand is elastic, and
weekend
Average Average demand is inelastic.
vertical merger?
D. The regular weekday
demand is inelastic, and A. A grocery store
weekend demand is buying another grocery
elastic. store in
the same market.

[124] Source: CMA 0697 B. A grocery store


1-17 buying another grocery
Patents are granted in store in a
order to encourage firms state where the first
to invest in company does not do
the research and business.
development of new
products. Patents are C. A hot dog producer
an example of buying a soft drink
manufacturer.
A. Vertical integration.
D. A brewer buying a
B. Market glass company.
concentration.

C. Entry barriers. [126] Source: Publisher


The output and cost
D. Collusion. information for a firm is
presented
below.
[125] Source: CMA 0697
1-18 Output Total Variable
Which one of the Cost Total Cost
following examples best ------ -------------------
describes a ----------
0 $-0- and capital in any desired
$100 proportions to produce
1 $150 its
$250 product. If wages rise to
2 $260 $20 per hour and capital
$360 equipment rentals rise to
3 $350 $22 per hour, in the long
$450 run the
The marginal cost of the firm will
second unit of output is
A. Use relatively more
A. $100 equipment and relatively
less
B. $110 labor in its production.

C. $150 B. Use relatively more


labor and relatively less
D. $180 equipment in its
production.

[127] Source: Publisher C. Use less of both


A company operating in a labor and equipment in
perfectly competitive its
market has production, but in the
been paying $10 per hour same proportions as
for labor and $20 per before.
hour to
rent a piece of capital D. Lower its average
equipment. The firm can equipment-output ratio
use labor and
raise its average labor-
output ratio.
conformity with generally
accepted accounting
[128] Source: Publisher principles:
If the price elasticity of
demand for a normal A B
good is C
estimated to be 2.5, a 5% --------- ---------
reduction in its price ---------
causes Sales $100,000
$200,000 $400,000
A. Total revenue to fall Cost of goods sold
by 5%. 60,000 120,000
200,000
B. Total revenue to fall Gross profit 40,000
by 12.5%. 80,000 200,000
Other expenses
C. Quantity demanded 10,000 20,000
to rise by 12.5%. 80,000
--------- ---------
D. Quantity demanded ---------
to decrease by 5%. Net income $ 30,000
$ 60,000 $120,000
=========
[Fact Pattern #5] ======== ========
Companies A, B, and C Shareholders'
had the following results equity $500,000
for last $300,000 $900,000
year as reported on
financial statements [129] Source: Publisher
prepared in (Refers to Fact Pattern
#5)
Assets are equal to
shareholders' equity. The C. Company C.
company has
no long-term debt D. Two were equal.
outstanding. The cost of
internally-generated
equity capital is 12%. [131] Source: Publisher
Which company Below are the grocers'
had the highest economic demand schedules for
profit? Palm Valley
Grapes. Assuming that
A. Company A. John, Towny, and
Dorothea are the
B. Company B. only three customers of
Palm Valley Grapes,
C. Company C. which of the
following sets of prices
D. Cannot be and output levels will be
determined from on the
information given. market demand curve?

Price John Towny


[130] Source: Publisher Dorothea
(Refers to Fact Pattern of Grapes Q Q
#5) Q
Which company had the dx dx dx
highest accounting --------- ---- ----- --------
income? $6 0 1 0
5 1 2 0
A. Company A. 4 2 4 0
3 3 6 1
B. Company B. 2 4 8 2
1 5 9 3 [133] Source: Publisher
A. ($6, 2); ($1, 17) The amount of
boysenberries demanded
B. ($5, 3); ($1, 17) for the third
quarter rose from 1,250
C. ($4, 4); ($2, 12) units to 1,750 units from
last year.
D. ($4, 0); ($1, 9) This was due to a
decrease in price from
$1.25 to $0.75
[132] Source: Publisher per unit. Therefore, the
Last week, the quantity price elasticity of
of apples demanded fell boysenberries is
from
51,500 units per week to A. 1/3
48,500 units per week. If
this B. 3/2
was a result of a 10%
price increase, what is C. 1
the price
elasticity of demand for D. 2/3
apples?

A. 1.67 [134] Source: Publisher


Suppose the price of
B. 1.06 mood rings rises from $3
to $4 when
C. 0.16 the quantity demanded of
mood rings decreases
D. 0.60 from
1,000 to 900. What would
be the price elasticity of
demand coefficient?

A. 3.00 [136] Source: Publisher


If Tonya used $10,000
B. 2.71 from her savings
account, which
C. 0.37 was paying 5% interest
annually, to invest in a
D. 0.33 saloon, the
opportunity cost of this
investment would
[135] Source: Publisher annually be
Demand price is inelastic
in which range of the A. $500
following
demand schedule? B. The dividend paid by
the coffee shop.
Price Quantity
Demanded C. $10,000
----- -----------------
$22 100 D. $10,500
18 200
14 400
10 600 [Fact Pattern #6]
6 800 Gator Company is selling
A. $22 - $18 in a purely competitive
market
B. $18 - $14 and has the following
cost data.
C. $14 - $10
Average Average
D. $10 - $6 Average
Fixed Variable If the market price for
Total Marginal Gator's product is $190,
Output Cost Cost this firm
Cost Cost should produce
------ ------- -------- --------
-------- A. 5 units at an
1 $600 $100 economic loss of $70.
$700 $100
2 300 75 B. 6 units and break
375 50 even.
3 200 70
270 60 C. 8 units and break
4 150 73 even.
223 80
5 120 70 D. 8 units at an
190 110 economic profit of $74.
6 100 90
190 190
7 86 105 [138] Source: Publisher
191 200 (Refers to Fact Pattern
8 76 119 #6)
195 230 If the market price for
9 67 138 Gator's product is $290,
205 290 this firm
10 60 160 should produce
220 360
A. 7 units at an
[137] Source: Publisher economic profit of $707.
(Refers to Fact Pattern
#6) B. 8 units at an
economic profit of $760.
C. 9 units at an
economic profit of $765. C. A shortage of 20
units.
D. 10 units at an
economic profit of $700. D. A shortage of 10
units.

[139] Source: Publisher


(Refers to Fact Pattern [141] Source: Publisher
#6) While walking around a
An equilibrium price will yard sale, Jimbo
be set at happened upon a
bicycle that he personally
A. $110 valued at $55. Jimbo was
able to
B. $190 purchase the bicycle for
$30, even though it was
C. $200 only 2
years old and originally
D. $230 sold for $100. What is
Jimbo's
consumer surplus?
[140] Source: Publisher
(Refer to Figure 1.) If a A. $70
legal price floor of $3.00
is B. $15
declared in the diagram,
what will be the result? C. $45

A. A surplus of 20 units. D. $25

B. A surplus of 10 units.
[Fact Pattern #7]
Holly, a horseshoe maker, A. $30
has collected the
following data B. $33
regarding the local
market for full sets of C. $36
horseshoes.
D. $39
Horseshoe Sets
Horseshoe Sets
Price Demanded [143] Source: Publisher
Supplied (Refers to Fact Pattern
----- -------------- -------------- #7)
$30 400 At each price, there is a
180 60 unit decrease in the
33 375 number of
250 horseshoes supplied for
36 350 every increase in the
290 cost of labor.
39 320 Therefore, the market
320 has a new equilibrium
42 285 price for
345 horseshoes of
45 235
395 A. $36

[142] Source: Publisher B. $39


(Refers to Fact Pattern
#7) C. $42
The market has an
equilibrium price for D. $45
horseshoes of
[144] Source: Publisher A. Perfectly inelastic.
(Refers to Fact Pattern
#7) B. Elastic.
Assume the supply stays
as shown in the previous C. Unit elastic.
table,
and the quantity of D. Inelastic.
horseshoes demanded at
each price
increase by 160 units. [146] Source: Publisher
What will be the new Suppose that a stairway
equilibrium manufacturer's price
quantity? elasticity of
demand was inelastic. If
A. 510 this manufacturer
decided to
B. 480 increase the price of its
stairways, what should
C. 445 have been
the result?
D. 395
A. Total revenues
decreased.
[145] Source: Publisher
When a 5% fall in the B. Total revenues
price of velcro shoes increased.
causes the
quantity demanded to C. Total revenues
increase by 10%, the remain unchanged.
demand for
velcro shoes is said to be
D. Total revenues were 2,000 $44 8,000
perfectly inelastic. 2,500 42 7,000
3,000 40 6,000
3,500 38 5,000
[147] Source: Publisher 4,000 36 4,000
(Refer to Figure 8.) In the 4,500 34 3,000
diagram, points J and K
have a [148] Source: Publisher
price elasticity of supply (Refers to Fact Pattern
between them of what? #8)
What would equilibrium
A. 1.33 price and quantity be?

B. .75 A. $40 and 6,000


pounds.
C. .40
B. $38 and 3,500
D. 2.50 pounds.

C. $36 and 4,000


[Fact Pattern #8] pounds.
Rock Salt, Inc. has
collected the following D. $34 and 3,000
information pounds.
regarding the current
market for rock salt.
[149] Source: Publisher
Salt Salt (Refers to Fact Pattern
Supplied #8)
Demanded Price What would occur if a
(pounds) legal price floor were set
-------- ----- ------------- at $42?
consumption of product
A. Shortage of 6,000 X.
pounds.
B. Increase
B. Surplus of 6,000 consumption of product Y
pounds. and increase
consumption of product
C. Shortage of 4,500 X.
pounds.
C. Increase
D. Surplus of 4,500 consumption of product X
pounds. and decrease
consumption of product
Y.
[150] Source: Publisher
Suppose a customer is D. Make no change in
deciding between consumption of product X
purchasing or
product X and product Y. Y.
The marginal utility of
product X
is 30 and its price is $10. [Fact Pattern #9]
The marginal utility of The following table
product Y shows the marginal-utility
is 45 and its price is $20. schedules for
In agreement with the goods M and N for a
utility-maximizing rule, consumer. The price of
the consumer should good M is
$2, and the price of good
A. Increase N is $4. The income of
consumption of product Y the
and decrease consumer is $18.
[152] Source: Publisher
Good M Good N (Refers to Fact Pattern
-------------- -------------- #9)
Quantity MU Quantity If the consumer wishes
MU to maximize utility, then
-------- -- -------- -- total utility
1 8 1 10 will equal
2 7 2 8
3 6 3 6 A. 24
4 5 4 4
5 4 5 3 B. 36
6 3 6 2
7 2 7 1 C. 48
8 1 8 0
D. Cannot be
[151] Source: Publisher determined.
(Refers to Fact Pattern
#9)
If the consumer wishes [153] Source: Publisher
to maximize utility, then (Refers to Fact Pattern
the #9)
consumer will purchase If the consumer's income
is increased from $18 to
A. 7M and 1N. $24 and
the consumer wishes to
B. 5M and 2N. maximize utility, then the
consumer
C. 3M and 3N. will purchase

D. 1M and 4N. A. 6M and 3N.

B. 8M and 2N.
C. 2M and 5N.
[155] Source: Publisher
D. 4M and 4N. Jim is satisfying his
hunger by consuming two
desserts, pies
[154] Source: Publisher and cakes. If the
The prices of A and B are marginal utility of a pie is
$5 and $4, respectively. half that of a
The cake, what is the price of
consumer is spending her a pie if the price of a
entire income buying 5 cake is
units of A $8.00?
and 7 units of B. The
marginal utility of both A. $4.00
the fifth unit of
A and the seventh unit of B. $8.00
B is 3. It follows that
C. $12.00
A. The consumer is in
equilibrium. D. $16.00

B. The consumer should


buy more of A and less of [Fact Pattern #10]
B. Following are total utility
data for tapes and CDs.
C. The consumer should Assume
buy less of A and more of that the prices of tapes
B. and CDs are $9 and $12,
respectively, and that
D. The consumer should consumer income is $54.
buy less of both A and B.
Units of Units
of [157] Source: Publisher
Tapes Total Utility (Refers to Fact Pattern
CDs Total Utility #10)
-------- ------------- -------- What is the consumer's
------------- total utility at the
1 9 1 equilibrium point?
16
2 15 2 A. 47
28
3 19 3 B. 51
36
4 21 4 C. 55
40
5 22 5 D. 61
42

[156] Source: Publisher [158] Source: Publisher


(Refers to Fact Pattern (Refers to Fact Pattern
#10) #10)
How many of each If the price of tapes
product will the decreases to $6, then the
consumer buy? consumer
will purchase
A. 1 tape and 4 CDs.
A. 4 tapes and 2 CDs.
B. 2 tapes and 2 CDs.
B. 2 tapes and 4 CDs.
C. 2 tapes and 3 CDs.
C. 3 tapes and 3 CDs.
D. 3 tapes and 3 CDs.
D. 4 tapes and 4 CDs.
For a firm, capital costs
$10 per unit and labor
[159] Source: Publisher costs $5
If a firm's fixed costs are per unit. If the marginal
$500, and its average product of labor is 10 and
variable the
costs stay constant marginal product of
despite various levels of capital is 5, the firm
output, which should
of the following must be
true? A. Employ less capital
and more labor.
A. Marginal cost will
equal average total cost. B. Employ more capital
and less labor.
B. Marginal cost will be
less than average C. Employ less capital
variable and labor.
cost.
D. Employ more capital
C. Average total cost and labor.
will decrease when
output is
increased. [161] Source: Publisher
Ekin Inc. has always used
D. Average total cost 100 units of capital and
will be constant. 100
units of labor to produce
10 automobiles.
[160] Source: Publisher Recently, Ekin
has decided to double outputs from 10 units to
both inputs. This 18 units. Microhard is
increase in experiencing
production has resulted
in the production of 15 A. Increasing returns.
more
automobiles. Ekin is B. Decreasing returns.
experiencing
C. Economies of scale.
A. Decreasing returns.
D. Constant returns.
B. Constant returns.

C. Increasing returns. [163] Source: Publisher


During the previous year,
D. Diseconomies of Morrison Inc. produced
scale. 200,000
pogo sticks and sold
them all for $10 each.
[162] Source: Publisher The explicit
Microhard Technologies costs of production were
has always used 100 $700,000, and the
units of implicit costs
labor and 100 units of of production were
capital to produce 10 $200,000. The firm had
keyboards. an accounting
Recently, Microhard has profit of
decided to double both
inputs. A. $1.1 million and
This increase in economic profit of $0.
production has resulted
in increasing the
B. $1.3 million and (Refers to Fact Pattern
economic profit of $1.1 #11)
million. Civic has an average
variable cost when 4
C. $1.3 million and units of output
economic profit of $1.3 are produced of
million.
A. $150
D. $1.3 million and
economic profit of $1.5 B. $200
million.
C. $250

[Fact Pattern #11] D. $600


The fixed cost of Civic
Co. is $1,000, and Civic's
total [165] Source: Publisher
variable cost is indicated (Refers to Fact Pattern
in the table. #11)
Civic has an average
Output Total Variable total cost when 4 units of
Cost output are
------ ------------------- produced of
1 $ 200
2 360 A. $150
3 500
4 600 B. $250
5 1,000
6 1,800 C. $400
7 2,800
D. $600
[164] Source: Publisher
B. $50,000
[166] Source: Publisher
(Refers to Fact Pattern C. $50,003
#11)
Civic's marginal cost of D. $80,000
the seventh unit of output
is
[168] Source: Publisher
A. $100 Regardless of output, a
firm has $4,000 a year in
B. $300 total fixed
costs. This same firm has
C. $400 an average variable cost
of $3,
D. $1,000 while producing 1,000
units of output. If the firm
decides to
[167] Source: Publisher produce 1,000 units,
When a firm produces what will be its average
10,000 units of output, its total cost?
total
variable cost is equal to A. $1.00
$50,000. Also, it
experiences B. $3.00
average fixed costs of $3
per unit. What are the C. $4.00
total costs
for producing 10,000 D. $7.00
units?

A. $30,000 [169] Source: Publisher


A firm produces only 5 maximize profits, Mr.
units of output. If total Bojangles will
variable cost recommend that the
is $400, and total fixed firm should
cost is $200, then
A. Not change output.
A. Marginal cost is This firm is at its
$120. profit-maximizing
position.
B. Average total cost is
$600. B. Decrease production.

C. Average fixed cost is C. Increase production.


$200.
D. Shut down.
D. Average variable
cost is $80.
[171] Source: Publisher
Mrs. Robinson is hired as
[170] Source: Publisher a consultant to a firm
Mr. Bojangles is hired as that is
a consultant to a firm currently competing
that is, perfectly. At the current
currently, competing output level
perfectly. At the current the price is $10, the
output level average variable cost is
the price is $20, the $6, the
average variable cost is average total cost is $10,
$15, average and marginal cost is $8.
total cost is $22, and To
marginal cost is $20. In
order to
maximize profits, Mrs. A. Not change the
Robinson will recommend output level, because the
that the monopolist is currently
firm should at the profit-maximizing
output
A. Decrease production. level.

B. Increase production. B. Shut down.

C. Shut down. C. Increase production.

D. Not change D. Decrease production.


production.

[173] Source: Publisher


[172] Source: Publisher The individual purely
As of December 31 of the competitive firm faces a
current year, a demand
monopolist was schedule that is
producing at a level that
experienced price = $18, A. Perfectly inelastic.
average
total cost = $15, average B. Inelastic but not
variable cost = $12, perfectly inelastic.
marginal
revenue = $13, and C. Perfectly elastic.
marginal cost = $14. To
maximize D. Elastic but not
profits in the new year, perfectly elastic.
the monopolist should

[174] Source: Publisher


Spruce Goose Inc. is a At its current production,
chief competitor in the Abba Co., a monopolist,
highly has a
competitive field of small marginal cost of $18, and
jet production. Its total marginal revenue of $21.
revenue Abba
for producing 10 small will maximize profits by
jets is $60. Meanwhile,
its total A. Increasing price
revenue for producing 11 while keeping production
small jets is $77. constant.
Therefore, the
B. Decreasing price and
A. Average revenue for increasing production.
producing 11 units is $17.
C. Decreasing both
B. Average revenue for price and production.
producing 11 units is $77.
D. Increasing both price
C. Marginal revenue for and production.
producing the eleventh
unit is
$17. [Fact Pattern #12]
Demand and cost data for
D. Marginal revenue for Billingsworth, a pure
producing the eleventh monopolist, is shown
unit is below.
$77.
Output Price per Unit
Total Cost
[175] Source: Publisher ------ -------------- ----------
1 $1,000 $ As a profit-maximizing
500 monopolist, Billingsworth
2 600 520 will sell its
3 500 580 product at a price of
4 400 700
5 300 A. $1,000
1,000
6 200 B. $600
1,500
C. $500
[176] Source: Publisher
(Refers to Fact Pattern D. $400
#12)
Billingsworth, a profit-
maximizing monopolist, [178] Source: Publisher
will produce (Refers to Fact Pattern
how many units? #12)
Billingsworth has decided
A. 1 to put 4 units on the
market. If
B. 2 Billingsworth is able to
sell each unit at the
C. 3 maximum price
that the buyer is willing
D. 4 to purchase it for, how
much would
total revenue be?
[177] Source: Publisher
(Refers to Fact Pattern A. $1,600
#12)
B. $2,500
C. $2,800 9 250
1,540
D. $4,000
[179] Source: Publisher
(Refers to Fact Pattern
[Fact Pattern #13] #13)
Soggy Shoes, a pure The profit-maximizing
monopolist, has output and price for
accumulated the Soggy Shoes is
following cost data.
A. 5 units and a $450
Quantity price.
Demanded Price per
Unit Total Cost B. 6 units and a $400
-------- -------------- ---------- price.
1 650
410 C. 7 units and a $350
2 600 price.
450
3 550 D. 8 units and a $300
510 price.
4 500
590
5 450 [180] Source: Publisher
700 (Refers to Fact Pattern
6 400 #13)
840 Assume Soggy Shoes is
7 350 able to engage in price
1,020 discrimination and sell
8 300 each unit of the product
1,250 at a price
equal to the maximum C. 8 units.
price the buyer of that
unit would be D. 9 units.
willing to pay. The
marginal revenue that
Soggy Shoes [182] Source: Publisher
obtains from the sale of (Refers to Fact Pattern
an additional unit will #13)
equal its Assume Dry Toes is a
nondiscriminating
A. Total cost. monopolist and
Soggy Shoes is a
B. Average cost. discriminating
monopolist. How much
C. Unit cost. greater of a total
economic profit will
D. Price. Soggy Shoes obtain
than Dry Toes, based
upon the data given for
[181] Source: Publisher Soggy
(Refers to Fact Pattern Shoes?
#13)
Assuming Soggy Shoes is A. $990
a price-discriminating
monopolist, what would B. $1,400
their profit-maximizing
output be? C. $1,560

A. 6 units. D. $2,550

B. 7 units.
[183] Source: Publisher
Suppose the price of a adds _______________ to the
factor of production is firm's profit, so
$10, and the ______________ labor should
product's price be hired.
(evaluated in a
competitive market) is A. $ 2; more.
$8. If
the last unit of a factor of B. $ 2; less.
production employed has
a C. $18; more.
marginal physical
product of 12, then this D. $18; less.
factor's marginal
revenue product is
[185] Source: Publisher
A. $96 A particular piece of
equipment, owned by
B. $24 Meehan Inc., is
to become worthless in
C. $10 exactly 1 year, the same
time in
D. $8 which it will produce its
last marginal revenue
product,
[184] Source: Publisher valued at $50,000. If the
The last hour of labor, interest rate is 8%, a firm
hired for $20, produces 6 would
units of be willing to buy the
output selling for $3 per piece of equipment when
unit. Therefore, that the purchase
labor-hour price is
A. Below $50,000. C. Four workers.

B. Below $46,296. D. Five workers.

C. Above $46,296.
[187] Source: Publisher
D. Above $50,000. (Refers to Fact Pattern
#14)
If the wage rate is $35,
[Fact Pattern #14] the firm will employ

Number of Total A. Two workers.


Product
Workers Production B. Three workers.
Price ($)
--------- ---------- --------- C. Four workers.
1 16 4
2 26 4 D. Five workers.
3 34 4
4 40 4
5 44 4 [188] Source: Publisher
(Refers to Fact Pattern
[186] Source: Publisher #14)
(Refers to Fact Pattern Assume the product has
#14) an increase to a fixed
If the wage rate is $15, price of $6,
the firm will employ then if the wage rate is
maintained at $35, the
A. Two workers. firm will
employ
B. Three workers.
A. Two workers.
$5. Therefore, the fourth
B. Three workers. unit of the resource will
have a
C. Four workers. marginal revenue product
of
D. Five workers.
A. $3

[Fact Pattern #15] B. $5

Units of Resource Total C. $10


Product Marginal
Product D. $15
----------------- -------------
----------------
1 8 [190] Source: Publisher
8 (Refers to Fact Pattern
2 14 #15)
6 Assume that the firm's
3 18 product still has a price
4 of $5 and
4 21 that the price of the
3 resource is valued at $20.
5 23 It can be
2 concluded that the firm
will employ _____ units of
[189] Source: Publisher the
(Refers to Fact Pattern resource.
#15)
The product that the firm A. 1
produces has a selling
price of B. 3
product of labor of 10 and
C. 4 a price of labor equal to
$5.
D. 5 Santa Maria's inputs also
have a marginal price of
capital of
[191] Source: Publisher 45 and a price of capital
At current conditions, the of $15. Assuming the firm
firm can sell 15 units of wishes
the to maximize profits,
output at a price equal to Santa Maria should
$1 per unit or 18 units of
the A. Use more labor and
output at a price equal to less capital.
$0.80 per unit. Therefore,
the B. Use less labor and
marginal revenue product less capital.
is
C. Use less labor and
A. $15.00 more capital.

B. $14.40 D. Make no change in


resource use.
C. $.60

D. $.60 [193] Source: Publisher


Pinta Inc., a profit-
maximizing
[192] Source: Publisher manufacturer, employs
Santa Maria Co. employs two
inputs that have a
marginal
resources, resource P
and resource Q. The price
of
resource P is $40 and its
marginal revenue product
(MRP)
for the last unit of P hired
was $240. The price of
resource
Q is $25 and its MRP for
the last unit of Q hired
was
$150. Pinta should

A. Hire more of
resource X and less of
resource Y.

B. Hire less of resource


X and more of resource
Y.

C. Hire less of both


resource X and resource
Y.

D. Hire more of both


resource X and resource
Y.
PART 1 A earned). If selling price
MICROECONOMICS drops below average
(ANSWERS) variable
cost, variable costs as
[1] Source: CMA 1285 1- well as fixed costs are no
15 longer being met, and
the firm should shut
Answer (A) is incorrect down.
because assuming price
exceeds average Answer (C) is incorrect
variable costs, a because in pure
shutdown would competition
result in the loss of the a single firm has no
contribution margin. influence on price.
Since
fixed costs are not Answer (D) is incorrect
avoided by shutting because this could only
down, the firm be
should continue to accomplished in the
produce in the short run long run.
as long as
it covers its variable
costs. [2] Source: CMA 1285 1-
16
Answer (B) is correct.
In the short run, a firm Answer (A) is incorrect
should because no specific time
continue operating as frame is contemplated
long as selling price by the definition.
exceeds
average variable cost Answer (B) is incorrect
(a contribution margin is because no specific time
frame is contemplated not refer to a minimum
by the definition. or maximum.

Answer (C) is incorrect Answer (B) is incorrect


because top because in this context,
management is the
but one of many inputs. term marginal means
The long run is the period incremental change and
within which all inputs does
can be varied. not refer to a minimum
or maximum.
Answer (D) is correct.
In the long run, no Answer (C) is incorrect
resource because marginal utility
costs are fixed. A firm is the
can therefore vary the change in total utility
amount from consumption of an
of any of its inputs. In additional unit of the
the short run, one or good.
more
inputs are fixed. Answer (D) is correct.
Utility is a measure of the
usefulness, or
[3] Source: CMA 1285 1- satisfaction, that results
17 from
consumption. Marginal
Answer (A) is incorrect utility is the amount of
because in this context, increase in utility when
the one additional unit is
term marginal means consumed. Marginal
incremental change and utility diminishes with
does additional
consumption. apply in the short run
because many costs are
fixed.
[4] Source: CMA 1286 1-4 In the long run, all
costs are variable.
Answer (A) is incorrect
because economies or Answer (D) is incorrect
diseconomies of scale because the law of
do not affect the demand diminishing returns
curve. describes the short-run
phenomenon in which,
Answer (B) is incorrect beyond some point,
because economies or increasing
diseconomies of scale amounts of a variable
do not affect the demand input will contribute less
curve. and
less to the total
Answer (C) is correct. product.
Economies and
diseconomies
of scale affect a [5] Source: CMA 1286 1-7
company's costs. When it
experiences economies Answer (A) is correct. If
of scale, a company's the market rate is less
long-run average costs than
will decrease. the maximum allowed,
Diseconomies a rent control law will
of scale occur when the have no
long-run average costs effect whatsoever.
increase. Economies or
diseconomies of scale do
not
Answer (B) is incorrect purchases are a result,
because if the higher not a determinant, of
price income
were required to be distribution.
charged, consumers
would Answer (C) is correct.
demand less housing Distribution of income is
and a surplus would determined in large
occur. part by ownership of the
factors
Answer (C) is incorrect of production: land,
because if price labor, capital, and
increases, management.
supply will increase. For example, a union
that monopolizes the
Answer (D) is incorrect labor
because demand will fall supply in an industry
when price increases. may be able to increase
the
incomes of its
[6] Source: CMA 0687 1-1 members. However, other
factors may
Answer (A) is incorrect also play roles. These
because transfer include inherent ability,
payments access
provide only a small to education or
portion of income. training, political
influence, and even
Answer (B) is incorrect sheer luck or the lack
because household thereof.
Answer (D) is incorrect Answer (C) is incorrect
because studies have because average costs
shown fall
that pretax and after- for any product whose
tax incomes are similarly production involves fixed
distributed. Thus, costs, such as natural
taxation in practice does monopolies.
not
achieve significant Answer (D) is correct.
income redistribution. Economies of scale exist
when a larger output is
associated with a lower
[7] Source: CMA 0687 1- average cost. Since
12 only large firms can
efficiently
Answer (A) is incorrect operate in such
because supply curves circumstances, the
are natural barriers to
normally assumed to be entry are substantial.
upward sloping for most Natural monopolies are
products. The supply those in
curve of a monopolist which significant
may be economies of scale are
essentially flat or even present.
downward sloping. Electric utilities are
examples of such
Answer (B) is incorrect companies.
because demand curves
are
normally downward [8] Source: CMA 1287 1-2
sloping for all products.
Answer (A) is incorrect percentage change in
because the increase in quantity demanded
price divided by
would decrease the the percentage change
quantity demanded. in price. If the result is
greater
Answer (B) is incorrect than one, demand is
because the price elastic, and total revenue
increase will
would not affect the increase with a
position of the demand decrease in price or
curve. decline with an
increase.
Answer (C) is incorrect
because the price
increase [9] Source: CMA 1287 1-5
would not affect the
position of the demand Answer (A) is incorrect
curve. because in pure (perfect)
competition, a
Answer (D) is correct. particular firm will
Given a relatively elastic produce for a
demand curve, an market in which the
increase in the price, selling price is given; only
such as from costs
a tax increase, would (input prices) can
lead to a decrease in influence the level of
expenditures. Price production.
elasticity of demand
equals the Answer (B) is incorrect
because in pure (perfect)
competition, a (input prices) can
particular firm will influence the level of
produce for a production.
market in which the
selling price is given; only
costs [10] Source: CMA 1287 1-
(input prices) can 10
influence the level of
production. Answer (A) is incorrect
because the short run
Answer (C) is correct. A can be
competitive firm will more or less than a
produce at a level at year depending upon a
which marginal cost firm's
equals ability to change its
marginal revenue inputs.
(selling price). Thus, a
change in Answer (B) is correct.
input prices affects The short run is defined
marginal cost and the as a
quantity of period so brief that a
output supplied. firm has insufficient time
to vary
Answer (D) is incorrect the amount of all
because in pure (perfect) inputs. Thus, the quantity
competition, a of one or
particular firm will more inputs is fixed.
produce for a The long run is a period
market in which the long
selling price is given; only enough that all inputs,
costs including plant capacity,
can be
varied. The firm will Answer (B) is incorrect
thus use both fixed and because regulation is
variable necessary when
inputs in the short run. demand for the natural
monopoly's
Answer (C) is incorrect product is inelastic.
because economies of Without regulation, the
scale monopolist could
are associated with the exploit consumers
long run. because they
would have to buy the
Answer (D) is incorrect product regardless of
because economic profits price.
may be earned, either
in the long or short run, Answer (C) is correct. A
when a natural monopoly is an
firm earns more than industry, such as a
the profits needed for it utility, in which
to consumers are
remain in operation. benefited by the
existence of a monopoly.
These
[11] Source: CMA 1287 1- monopolies are
12 regulated by the
government to
Answer (A) is incorrect control prices and
because in a natural prevent abuses. The
monopoly, marginal reason for a
costs continue to natural monopoly is
decrease. that economies of scale
are so
great that the lowest Answer (A) is incorrect
average cost is reached because perfect
when inelasticity
only one firm is in the exists when the
industry. Fixed costs are quantity demanded is
so constant for all
high that relatively low prices (the coefficient
prices are possible only of elasticity is zero).
at high
levels of sales. If Answer (B) is correct.
competition existed, The coefficient of
each firm's elasticity
share of the market measures the
would be insufficient to responsiveness of the
drive percentage
down unit costs enough change in quantity
to permit charging a low demanded (the
price. numerator) to a
given percentage
Answer (D) is incorrect change in the price of a
because consumer product (the
demand denominator).
would be inelastic and Consumer demand is said
not influenced by to be elastic
business when the coefficient of
cycles. elasticity is greater than
one.

[12] Source: CMA 1288 1- Answer (C) is incorrect


22 because inelastic
demand is
characterized by a
coefficient of less than Answer (C) is incorrect
one. because demand is
inelastic
Answer (D) is incorrect when the coefficient is
because unitary less than one but greater
elasticity than
exists when the zero.
coefficient is exactly
one. Answer (D) is incorrect
because unitary
elasticity
[13] Source: CMA 1288 1- exists when the
23 coefficient is exactly
one.
Answer (A) is correct.
When the coefficient of
elasticity (percentage [14] Source: CMA 1288 1-
change in 27
demand/change in
price) is less than one, Answer (A) is incorrect
demand is inelastic. because it is an attribute
When the of
coefficient is zero, the perfect competition.
demand is perfectly
inelastic. Answer (B) is incorrect
because it is an attribute
Answer (B) is incorrect of
because demand is perfect competition.
elastic
when the coefficient is
greater than one.
Answer (C) is correct. which no economic
Perfect competition profits are earned.
assumes a
large number of buyers Answer (D) is incorrect
and sellers that act because it is an attribute
independently, a of
homogeneous or perfect competition.
standardized
product, free entry into
and exit from the market, [15] Source: CMA 1288 1-
perfect information, no 25
nonprice competition, no
control over prices Answer (A) is incorrect
(sellers are price takers because the increase in
rather income shifts the
than price setters), and demand, not the supply,
an equilibrium price curve.
equal to
the average total cost. Answer (B) is incorrect
Given free entry into the because moving down an
market and perfect existing demand curve
information, an increase is the effect of a lower
in profits price.
in the industry will The change in income
result in an increase in shifts the economy to a
the number new
of firms in the market. demand curve.
This increase will have
the Answer (C) is incorrect
long-run effect of because a shift to the left
reducing the price to the
level at
means that consumers when the price is
will buy fewer products changed. A shift in the
at curve itself
each price. This occurs when any of the
leftward shift in response determinants changes.
to Such
increased income is shifts can be caused by
characteristic of an a change in the tastes
inferior good, and
not a normal good. The preferences of
demand for inferior goods consumers toward a
is product, for
inversely related to example, as a result of
income. Hamburger is an a successful advertising
inferior campaign, an increase
good, and steak is a in consumer income (if a
normal good. product is a normal
good), or changes in the
Answer (D) is correct. prices of
The demand schedule is substitute or
a complementary products.
relationship between An increase in
the prices of a product consumer income
and the would shift the demand
quantity demanded at curve to
each price, holding other the right and result in
determinants of the greater consumption of
quantity demanded the
constant. A product at each price.
movement along an
existing demand curve
occurs
[16] Source: CMA 1288 1- profits, and price
29 exceeds marginal cost,
resulting in
Answer (A) is correct. an underallocation of
Monopolistic competition resources. Firms produce
assumes a large less
number of firms with than the ideal output,
differentiated and the industry is
(heterogeneous) populated by
products, and relatively too many firms that are
easy entry too small in size. Price is
into the market. Sellers higher and output less
have some price control than in pure competition.
because of product
differentiation. Answer (B) is incorrect
Monopolistic because responses to
competition is competitors' actions
characterized by are unnecessary if
nonprice competition, products are
such as advertising, sufficiently
service after the sale, differentiated to make
and price competition
emphasis on trademark meaningless.
quality. In the short run,
firms Answer (C) is incorrect
equate marginal because product
revenue and marginal differentiation permits
cost. In the some price control.
long run, firms tend to
earn normal (not Answer (D) is incorrect
economic) because the availability
of
close substitutes to barriers that can be
makes the product either natural, such as an
demand curve absolute cost
elastic. advantage, or artificial,
such as massive
advertising by existing
[17] Source: CMA 1288 1- firms. The price rigidity
30 normally found in
oligopolistic markets can
Answer (A) is incorrect be
because an oligopoly can explained in part by the
have either kinked demand curve
differentiated theory.
(automobiles) or Because competitors
undifferentiated (steel) respond to price changes
products. by
one of the firms in an
Answer (B) is correct. oligopolistic industry, the
An oligopoly consists of demand curve for an
only oligopolist's products
a few firms. Each firm tends to
reacts to decisions of be kinked. Price
other decreases are usually
firms in the industry. matched, with
Prices in a noncollusive no increase in market
market share. Price increases
tend to be rigid, or are often
sticky, because of the not followed, with a
interdependence loss of market share by
among firms. Entry is the first
difficult thanks
to raise prices. An
oligopoly must often [18] Source: CMA 0689 1-
confront 20
cyclical or seasonal
fluctuations in the Answer (A) is incorrect
quantity because economic
demanded. Price efficiency
rigidity makes it difficult is measured in dollar
for terms.
oligopolists to maintain
sales levels by reducing Answer (B) is incorrect
price because opportunity cost
when the demand curve is
shifts to the left or by defined as the return
increasing sales and possible from the next
output when demand best
increases. alternative other than
the one selected.
Answer (C) is incorrect
because the kinked Answer (C) is correct.
demand Technological efficiency
curve explains price is
rigidity in an oligopoly. defined as the
relationship between the
Answer (D) is incorrect physical
because an oligopoly can output of a given
have either technology and the
differentiated maximum output
(automobiles) or that is possible.
undifferentiated (steel)
products. Answer (D) is incorrect
because comparative
advantage compares within a single country.
the costs of producing
products Answer (D) is incorrect
within a single country. because absolute
advantage
compares input costs
[19] Source: CMA 0689 1- among countries.
21

Answer (A) is incorrect [20] Source: CMA 0689 1-


because economic 22
efficiency
is a comparison Answer (A) is incorrect
measured in dollar terms. because an equality of
costs
Answer (B) is correct. would provide no
Opportunity cost is the incentive or disincentive
benefit to
forgone by using scarce substitute.
resources in a given way.
Thus, it is the benefit Answer (B) is incorrect
that could have been because profits will be
obtained greater if a lower-cost
from the best substitute can be used.
alternative use of the
resources. Answer (C) is correct.
The principle of
Answer (C) is incorrect substitution
because comparative states that, when given
advantage compares a choice, the firm will
the costs of producing choose
products
the factor of production evaluated when
that is less expensive. In purchasing capital goods.
other words, the
cheaper input will be Answer (D) is correct. A
used as a firm considers such
substitute for the factors
higher-priced input. as price, relationships
with suppliers, product
Answer (D) is incorrect demand, avoidable
because the firm will not future costs, economies
knowingly use more of of scale,
an expensive input if a opportunity costs,
cheaper substitute can technological efficiency,
be used. and the
capital-labor ratio when
evaluating the purchase
[21] Source: CMA 0689 1- of
24 new equipment. The
cost of the current
Answer (A) is incorrect facility would
because this should be not be relevant
evaluated when because it is a sunk cost.
purchasing capital goods.

Answer (B) is incorrect [22] Source: CMA 0689 1-


because this should be 26
evaluated when
purchasing capital goods. Answer (A) is incorrect
because a shift to the left
Answer (C) is incorrect (a
because this should be
decrease in demand) price results in greater
would occur if the price demand by consumers at
of a each
complementary product price level. A movement
increased. along an existing demand
curve occurs when the
Answer (B) is incorrect price for the product is
because demand for a changed but demand is
substitute would constant.
decrease, thereby
lowering its
equilibrium price. [23] Source: CMA 0689 1-
29
Answer (C) is incorrect
because the demand Answer (A) is incorrect
curve because the supply curve
will shift to the right, will
but the supply curve will be unaffected, but the
not demand curve will shift
change. to the
right.
Answer (D) is correct. A
decrease in the price of a Answer (B) is incorrect
complementary good because the supply curve
(e.g., gasoline) will cause will
the be unaffected, but the
demand curve of the demand curve will shift
joint commodity (e.g., to the
automobiles) to shift to right.
the right (increase). The
lower Answer (C) is correct.
The relative price of the
normal good is Answer (A) is incorrect
improved relative to because revenue should
other goods as a rise
consequence of as the price drops.
inflation. Thus,
consumers will Answer (B) is incorrect
demand more of the because revenue should
normal good. This rise
increase in as the price drops.
demand is caused not
by a price change but by Answer (C) is correct.
a Elasticity is the change in
favorable change in the demand divided by the
preferences of change in price. An
consumers. As elasticity
a result, the demand of 1.5 means that the
curve for the product will change in demand (the
shift to numerator of the
the right. fraction) will increase by
150% of
Answer (D) is incorrect any change in price
because the supply curve (the denominator)
will measured in
be unaffected, but the absolute terms (the
demand curve will shift minus sign is ignored).
to the Thus, a
right. 10% price reduction
increases demand by
15% (1.5
[24] Source: CMA 1289 1- x 10%).
4
Answer (D) is incorrect significant barriers to
because the price decline entry, such as high
will capital
lead to increased requirements, which
demand if elasticity is prevent new firms from
greater than entering
1.0. the industry. The
automobile industry is an
example.
[25] Source: CMA 1289 1-
11 Answer (D) is incorrect
because the demand
Answer (A) is incorrect curve
because an industry with for a firm in perfect
a competition is horizontal.
single firm is An
monopolistic. oligopolist's demand
curve is relatively flat but
Answer (B) is incorrect may be
because oligopolists kinked if competitors
have the follow its price decreases
same profit-maximizing but
goal as other firms. not the price increases.

Answer (C) is correct.


An oligopolistic industry [26] Source: CMA 0690 1-
is 19
characterized by only a
few firms. Usually there Answer (A) is correct. A
are normal good that
competes
with several similar decrease in price).
items will have an
elasticity Answer (C) is incorrect
greater than one (its because consumers
demand is relatively purchase
elastic) more (less) of a normal
because a given good (such as steak) at
percentage change in every
price will result price when income
in a greater percentage levels increase
change in sales as (decrease). The
consumers opposite is true of
shift to or from the inferior goods (such as
close substitutes. Price hamburger). These
elasticity of effects result because
demand is promoted by normal
the availability of goods have a positive
substitute income elasticity of
goods. demand
and inferior goods have
Answer (B) is incorrect a negative income
because demand has elasticity.
perfect The availability of close
price elasticity if a substitutes affects the
small change in price price
causes elasticity of demand
buyers to reduce their but not income elasticity.
purchases to zero (an
increase Answer (D) is incorrect
in price) or to purchase because inelasticity is
all of the available supply more
(a
apt to occur when few amount demanded, and
substitutes are available. total revenues will
increase.

[27] Source: CMA 0690 1- Answer (C) is incorrect


20 because the distinction
between inferior and
Answer (A) is incorrect normal goods is based on
because if demand is the
price effects on quantity
elastic, the quantity demanded of a change in
demanded will decline by income.
a
greater percentage Answer (D) is incorrect
than the percentage because perfect
price inelasticity
increase. means there would be
no decline in demand
Answer (B) is correct. because
An increase in gasoline of a price increase.
prices
during the summer
implies that demand for [28] Source: CMA 0690 1-
gasoline is 21
relatively price
inelastic in the short run. Answer (A) is incorrect
That is, the because no information is
price increase will given about the effects
result in little or no of changes in the price.
decline in the
Answer (B) is incorrect
because no information is
given about the effects percentage change in
of changes in the price. income.

Answer (C) is incorrect


because no information is [29] Source: CMA 0690 1-
given about the effects 22
of changes in the price.
Answer (A) is incorrect
Answer (D) is correct. because utility products
Consumers purchase are
more price inelastic.
(less) of a normal good
(such as steak) at every Answer (B) is incorrect
price because utility products
when income levels are
increase (decrease). The price inelastic.
opposite is true of
inferior goods (such as Answer (C) is correct.
hamburger). These Utility products are
effects result because typically
normal price inelastic because
goods have a positive they are regarded as
income elasticity of necessities for which
demand few substitutes are
and inferior goods have available.
a negative income That is why utilities are
elasticity. regulated by state
Income elasticity of agencies.
demand equals the The existence of
percentage monopolies and price
change in quantity inelastic
demanded divided by the
products would permit Answer (B) is incorrect
an unregulated utility to because the demand
take curve is
unfair advantage of downward sloping.
consumers.
Answer (C) is incorrect
Answer (D) is incorrect because a vertical
because utility products demand
are curve signifies that the
not perfectly price quantity demanded does
inelastic. Some not
consumers would change with price.
reduce their
consumption as prices Answer (D) is correct.
rise. Given perfect The demand curve for a
price inelasticity of consumer good (normal
demand, the amount or inferior) is downward
demanded sloping to the right. At
will be the same at all high prices, the amount
prices. demanded is relatively
low. As prices decrease,
the
[30] Source: CMA 0690 1- amount demanded
23 increases. The
substitution effect is
Answer (A) is incorrect the change in the cost
because the demand of a good relative to
curve is others that
downward sloping. will cause a cheaper
good to be substituted for
more
expensive ones. The would not affect the
income effect is the position of the demand
change in curve; it
purchasing power would only change the
experienced by equilibrium point at
consumers as a which the
result of a price change two curves intersect.
(real income increases or
decreases). Both of Answer (C) is correct. A
these effects cause the shift to the left means
price of a that
product and the the quantity demanded
quantity demanded to be will be less at each price.
inversely This
related. result follows because
cars and gasoline are
complementary goods.
[31] Source: CMA 0690 1- If automobile prices
24 increase,
fewer cars are
Answer (A) is incorrect purchased, fewer miles
because a price change are driven, and
causes a movement to the quantity of gasoline
a new point on the demanded declines at
demand each
curve, not a shift of the price.
curve itself.
Answer (D) is incorrect
Answer (B) is incorrect because a decrease in
because a change in car
supply prices would result in
the sale of more cars and
increased demand at some point beyond
each gasoline price. The which additional units of
effect the
would be a shift of the variable input will
demand curve to the contribute less and less
right. to the total
product. In other words,
marginal product will
[32] Source: CMA 0690 1- decline. An assembly
26 plant with lower unit
costs at
Answer (A) is incorrect 85% than at 95%
because it illustrates the capacity is experiencing
concept of increasing diminishing
returns to scale. returns.

Answer (B) is incorrect Answer (D) is incorrect


because it illustrates the because it illustrates the
concept of increasing concept of increasing
returns to scale. returns to scale.

Answer (C) is correct.


The law of diminishing [33] Source: CMA 0690 1-
returns 27
states that, when
successive equal Answer (A) is correct.
amounts of a Economic (pure) profit
variable input are equals
applied to a fixed input, total revenue minus
there is economic costs.
Economic costs
are defined by the former fails to
economists as include a deduction for
opportunity costs, opportunity
which are the values of costs, for example, the
productive resources in salary forgone by an
their entrepreneur who
best alternative uses. chooses to be self-
The return sufficient to employed.
induce
the entrepreneur to Answer (B) is incorrect
remain in business because both economists
(normal profit) and
is an economic cost. accountants treat
Net income as computed interest as a cost.
under
generally accepted Answer (C) is incorrect
accounting principles because economic profits
considers will be less than net
only explicit costs, not income.
such implicit costs as
normal Answer (D) is incorrect
profit and the because economic profits
opportunity costs will be less than net
associated with not income.
using assets for
alternative purposes.
Thus, net [34] Source: CMA 0690 1-
income will be higher 29
than economic profit
because Answer (A) is incorrect
because the marginal
revenue
curve is downward curve is downward
sloping in these cases. sloping in these cases.
Thus, the Thus, the
point on the MC curve point on the MC curve
equal to MR indicates the equal to MR indicates the
profit-maximizing profit-maximizing
quantity but not the quantity but not the
price. The price price. The price
is found by extending a is found by extending a
perpendicular line up to perpendicular line up to
the the
demand curve from the demand curve from the
MR = MC point. In other MR = MC point. In other
words, the price words, the price
(vertical) coordinate of (vertical) coordinate of
each point each point
on the MC curve does on the MC curve does
not equal the market not equal the market
price at price at
a given level of output a given level of output
except in pure except in pure
competition. In competition. In
this structure, the MR this structure, the MR
curve is the firm's curve is the firm's
demand demand
curve. curve.

Answer (B) is incorrect Answer (C) is correct.


because the marginal Under pure competition,
revenue the
portion of a firm's profit-maximizing
marginal cost (MC) curve quantity but not the
above price. The price
its average variable is found by extending a
cost curve (the firm perpendicular line up to
cannot cover the
its variable costs below demand curve from the
this shutdown point) is MR = MC point. In other
also words, the price
its supply curve. (vertical) coordinate of
Because the firm is a each point
price taker (the on the MC curve does
marginal revenue (MR) not equal the market
curve is horizontal), price at
increases a given level of output
in supply can occur except in pure
only at higher prices competition. In
because the this structure, the MR
firm produces at the curve is the firm's
level at which MR equals demand
MC. curve.

Answer (D) is incorrect


because the marginal [35] Source: CMA 0690 1-
revenue 30
curve is downward
sloping in these cases. Answer (A) is correct.
Thus, the Enhanced technology and
point on the MC curve worker productivity
equal to MR indicates the would cause the supply
curve to
shift to the right where cannot be determined
it would intersect with from the information
the given.
demand curve at a
lower price level. The
shift is [36] Source: CMA 1290 1-
caused by the 1
producer's ability to
lower costs and Answer (A) is correct.
produce more at a Economic or pure profit is
given market price. what remains after all
explicit and implicit costs
Answer (B) is incorrect have
because the increased been deducted from
productivity would total revenue. These
cause a rightward shift in costs
the include the opportunity
supply curve. costs of the use of
resources
Answer (C) is incorrect as well as the
because the rightward payments made to
shift others. Economic
would result in an profit is normal in a
intersection with the monopoly because a
demand curve monopolist
at a lower price. can bar the entry of
competitors and
Answer (D) is incorrect influence the
because the effect on market price.
wages
Answer (B) is incorrect Answer (A) is incorrect
because, by definition, a because rackets and
pure balls
monopoly consists of are not substitutes for
only one firm. Thus, no each other; both are
decline needed to
in the number of firms play tennis.
can occur unless the
monopolist goes out of Answer (B) is incorrect
business. because rackets and
balls are
Answer (C) is incorrect not independent goods;
because an increase in they are used together.
supply could lower
prices, a result that Answer (C) is incorrect
would not because an inferior good
benefit the monopolist. is
one that experiences
Answer (D) is incorrect increased (decreased)
because the demand for demand
and as consumers' incomes
prices of substitutes decline (rise). Tennis
might increase if the equipment is a leisure
monopolist item, not an inferior good.
charges a price
sufficient to earn an Answer (D) is correct.
economic profit. As sales of tennis rackets
increase, a
corresponding increase
[37] Source: CMA 1290 1- should occur in
2
the demand for tennis
balls because balls are Answer (D) is correct.
needed Pure competition is
to use the rackets. characterized by
Thus, the two items are numerous buyers and
complementary goods. sellers who
act independently, a
standardized product,
[38] Source: CMA 1290 1- ease of
3 entry into or exit from
the market, perfect
Answer (A) is incorrect information, the
because sellers in pure inability of each firm to
competition act influence
independently. prices, and the absence
of nonprice competition.
Answer (B) is incorrect
because the need for
large [39] Source: CMA 1290 1-
research and 4
development programs
would make it Answer (A) is incorrect
difficult for new because an increase in
participants to enter the supply causes a decline
market. in price if demand is
constant.
Answer (C) is incorrect
because competition is Answer (B) is incorrect
characterized by because an increase in
product similarity; demand and a decline
differentiation in supply both result in
reduces competition. higher
prices. [40] Source: CMA 1290 1-
5
Answer (C) is incorrect
because an increase in Answer (A) is correct. A
both government price support
demand and supply program will cause
results in a higher producers to supply more
equilibrium goods
quantity. than can be absorbed
by the market if the
Answer (D) is correct. support
An increase in demand price is higher than the
signifies a rightward equilibrium price. The
shift in the demand effect
curve, that is, will be surpluses
an increase in the because the amount
quantity demanded at supplied will
each price. A exceed the amount
decrease in supply demanded.
involves a leftward shift
in the Answer (B) is incorrect
supply curve, that is, a because no shortages
reduction in the quantity will
supplied at each price. occur. Suppliers will be
Each event increases the induced by the higher
equilibrium price if than
other factors are equilibrium price to
constant. Thus, if produce more than the
both events occur, the amount
price will increase. demanded.
Answer (C) is incorrect Answer (A) is incorrect
because no rationing because price ceilings
would are
occur. Consumers will essentially the opposite
be able to buy all they of price supports. If price
desire is
because supply will subject to a ceiling,
exceed demand. suppliers may receive
less than at
Answer (D) is incorrect the equilibrium level.
because firms will be But the supplier may
encouraged to enter receive
the industry by the more at the equilibrium
availability of rate if the price is
greater revenue than supported.
that provided by
consumer Answer (B) is incorrect
demand. In fact, price because prices may be
support programs are lower
often than at the equilibrium
designed with the level.
intention of keeping
marginal firms Answer (C) is correct.
from leaving the Government imposed
industry. price
ceilings may cause
prices to be set at rates
[41] Source: CMA 1290 1- below
6 equilibrium. Because
prices are set artificially
low,
demand will tend to number of firms, most
exceed the supply and are likely to be small,
shortages with
will occur. correspondingly low
economies of scale and
Answer (D) is incorrect capital
because price ceilings needs.
cause
shortages if consumers Answer (D) is correct.
demand more than Monopolistic competition
sellers are is
willing to supply. characterized by the
existence of a large
number of
[42] Source: CMA 1290 1- firms, differentiated
7 products, relative ease of
entry,
Answer (A) is incorrect some control of price
because entry is possible by the firms, and
and significant
relatively easy. Blocked nonprice competition
entry is typical of (e.g., by advertising).
monopoly. Entry into
monopolistic
Answer (B) is incorrect competition is more
because difficult entry is difficult than entry
typical of oligopoly. into pure competition,
but it is relatively easy
Answer (C) is incorrect compared with entry
because, given the large into a monopoly.
[43] Source: CMA 1290 1- Answer (D) is correct.
8 The long-run average
total
Answer (A) is incorrect cost curve is normally
because the curve is not U-shaped because the
horizontal. It varies average
with changes in cost per unit declines
production scale. as the firm experiences
economies of scale.
Answer (B) is incorrect However, beyond the
because, although point of
returns to intersection with the
incremental input may marginal cost curve, it
be diminishing and will begin
marginal rising because the
cost rising, average amount added (marginal
total cost may continue cost) to
to total cost is greater
decline. than the average total
cost.
Answer (C) is incorrect
because the average
total [44] Source: CMA 1290 1-
cost curve is a U- 10
shaped curve that is
intersected at Answer (A) is incorrect
its minimum point by because economic (pure)
the rising portion of the profit is the residual
marginal return in excess of
cost curve. normal profit.
Economic profit equals
total revenue minus
opportunity costs.
These are the sum of [45] Source: CMA 1290 1-
explicit and 9
implicit costs, including
normal profit. Answer (A) is incorrect
because labor
Answer (B) is incorrect specialization
because accounting means more efficient
profit is use of labor and lower
the excess of total costs.
revenue over explicit
costs Answer (B) is incorrect
(out-of-pocket because the ability to use
payments to outsiders). by-products results in
lower costs and thus
Answer (C) is incorrect economies of scale.
because a normal profit
is an Answer (C) is incorrect
implicit cost. because the fixed costs
related to the use of
Answer (D) is correct. capital equipment can be
Normal profit is the level spread
of over more products
profit necessary to when such equipment is
induce entrepreneurs to more
enter and efficiently used.
remain in the market.
Economists view this Answer (D) is correct.
profit as Economies of scale result
an implicit cost of in a
economic activity. decline in the average
cost of production. These
economies result from law of diminishing
specialization of labor returns assumes a fixed
and component
management, the of input.
ability to use by-products
and
scrap, and more [46] Source: CMA 1290 1-
efficient use of capital 11
equipment.
The law of diminishing Answer (A) is correct. A
returns states that, as natural monopoly occurs
additional units of a when economies of
variable input are scale are very great. In a
combined with natural
a fixed input, marginal monopoly, the unit cost
product will begin to fall of meeting the entire
when demand
the theoretical limit of for a product is
efficiency is exceeded. minimized when there is
Hence, only one firm
this principle is not in the industry. Thus,
applicable to the the presence of two or
discussion of more
economies of scale, firms would prevent the
which explains realization of the
diminishing economies
average cost. of scale necessary to
Moreover, the latter minimize cost. Public
concept assumes utilities are
that all costs are common examples of
variable in the long run, natural monopolies.
whereas the
Answer (B) is incorrect concentration ratio
because monopolistic indicates that few firms
competition is a market are
structure in which many operating in an
firms industry. Thus, a high
operate. ratio indicates
monopoly power.
Answer (C) is incorrect
because natural Answer (B) is incorrect
monopolies because a highly
exist when economies competitive
of scale are very great. industry would have a
low concentration ratio.
Answer (D) is incorrect
because natural Answer (C) is incorrect
monopolies because the
exist when economies concentration
of scale are very great. ratio has no
relationship to the law of
demand.
[47] Source: CMA 1290 1-
12 Answer (D) is incorrect
because a high
Answer (A) is correct. concentration
The concentration ratio ratio is more indicative
is the of a monopoly than of
percentage of total monopolistic
industry sales competition, which is
attributable to a characterized by
specified number of the the existence of many
largest firms. A high firms.
Answer (D) is incorrect
[48] Source: CMA 1290 1- because the supply curve
13 shows only the supply,
not the demand.
Answer (A) is correct. A
supply curve illustrates
the [49] Source: CMA 1290 1-
relationship between 14
price and the quantity of
a good Answer (A) is incorrect
that sellers are willing because agriculture is
to supply. As price closer
increases, to pure competition.
the supply will Many producers sell an
increase. undifferentiated
product.
Answer (B) is incorrect
because consumer tastes Answer (B) is correct.
are Monopolistic competition
reflected by the is
demand curve, not the best represented by the
supply curve. fast food industry. Many
firms are in
Answer (C) is incorrect competition, but the
because the demand products are
curve differentiated.
shows the relationship
between price and Answer (C) is incorrect
quantity because the steel
demanded. industry
has few firms and is
best characterized as an
oligopoly. Moreover, its the change in revenue
products tend to be from selling one
standardized. additional unit
of product, not from a
Answer (D) is incorrect change in prices.
because the auto Moreover, in
industry is market structures other
oligopolistic. Few than pure competition,
producers exist because marginal revenue
of the declines as sales
difficulty of entering increase because
the market. prices eventually must
be lowered to stimulate
demand.
[50] Source: CMA 1290 1-
15 Answer (C) is incorrect
because marginal
Answer (A) is incorrect revenue
because marginal cannot be greater than
revenue price.
equals price in pure
competition. In Answer (D) is correct.
monopolistic Marginal revenue is the
competition, no firm change in total revenue
can affect the price. resulting from producing
Hence, the and
price is the same at all selling one additional
levels of output. unit of product. Marginal
revenue is constant in
Answer (B) is incorrect pure competition but
because marginal declines
revenue is
as output rises in other Answer (D) is correct.
market structures. The kinked demand curve
Equating is
marginal revenue and characteristic of an
marginal cost is the oligopoly. This
profit-maximizing phenomenon
position for all firms. occurs because
decreases but not
increases tend to
[51] Source: CMA 1290 1- be matched by the
16 other firms in the
industry. Thus,
Answer (A) is incorrect the slope of the
because an agricultural demand curve is likely to
market approximates change
pure competition and significantly above and
exhibits a below the current price.
normal demand curve. An
oligopolist that raises
Answer (B) is incorrect its price is likely to lose
because monopolistic substantially more of
competition is its market share than it
characterized by a would
normal demand gain by a price cut. In
curve. other words, the curve is
relatively elastic with
Answer (C) is incorrect respect to price
because pure increases and
competition is less elastic for price
characterized by a cuts.
normal demand curve.
[52] Source: CMA 1290 1- greater than 1.0 if the
17 total revenue increases
as the
Answer (A) is correct. A result of a price
decline in price decrease.
accompanied by an
increase in total revenue Answer (D) is incorrect
indicates because the elasticity is
that quantity demanded greater than 1.0.
has increased by a
greater
percentage than the [53] Source: CMA 1290 1-
percentage price 18
decrease.
Hence, the price Answer (A) is incorrect
elasticity of demand is because the short-run
greater than supply
1.0. Demand is elastic curve is derived from
when it is greater than the marginal cost curve,
1.0. not the
average total cost
Answer (B) is incorrect curve.
because the increase in
revenue resulting from Answer (B) is incorrect
the price decrease because the short-run
indicates supply
elasticity. curve is derived from
the marginal cost curve,
Answer (C) is incorrect not the
because elasticity must fixed cost curve.
be
Answer (C) is correct. average variable cost
In the short run, certain will constitute the short-
costs run
are fixed regardless of supply curve. At a price
output. Given that fixed below the average
costs variable
are incurred even if the cost, the firm's losses
firm shuts down, the firm equal its fixed costs plus
gains in the short run some
by continuing to operate of its variable costs. In
if this case, the firm would
revenues exceed close
variable costs. down and not supply
Accordingly, the anything.
firm's short-run supply
curve is derived from the Answer (D) is incorrect
marginal cost curve. because the short-run
The firm benefits by supply
producing curve is derived from
until marginal cost the marginal cost curve,
equals marginal revenue. not the
As long total cost curve.
as the marginal cost is
lower than the marginal
revenue, the firm will [54] Source: CMA 0691 1-
recover some of its fixed 13
costs
as well as its variable Answer (A) is incorrect
costs. However, only the because the degree of
segment of the elasticity is not a
marginal cost curve that sufficient predictor of the
lies above nature of
the change in market elasticity is not a
price. sufficient predictor of the
nature of
Answer (B) is incorrect the change in market
because the degree of price.
elasticity is not a
sufficient predictor of the
nature of [55] Source: CMA 0691 1-
the change in market 14
price.
Answer (A) is incorrect
Answer (C) is correct. If because economic goods
demand is constant, an can be increased in
increase in supply will quantity if suppliers want
lower the price of a good. to
If supply more goods.
supply is constant, an
increase in demand will Answer (B) is incorrect
raise because the issue is not
the price. When both whether actual
events occur, the effect production is adequate
depends on the but whether
magnitude of the potential production
changes. Thus, the would be adequate to
market price is not meet
predictable from the zero-price demand.
facts given.
Answer (C) is correct.
Answer (D) is incorrect Economic goods are
because the degree of demanded by
consumers but exist in
insufficient
supply to satisfy all paid when the supply is
demands for them. Thus, perfectly inelastic.
economic goods are
scarce because, at a zero Answer (B) is correct.
price, Economic rent is the
demand would exceed total
the potential supply that price paid for land and
can other natural resources
be produced given the that
finite resources (inputs) have a completely fixed
available on this planet. supply. Given this perfect
The problem facing inelasticity, the sole
society is factor determining rent
how to allocate a fixed for land
supply of resources. and other resources
with a completely fixed
Answer (D) is incorrect total
because an economic supply is demand. A
good higher price will not
does not have to be increase
man-made. supply and thereby the
productive potential of
the
[56] Source: CMA 0691 1- economy. Thus, no part
15 of the rent paid for such
resources provides an
Answer (A) is incorrect incentive to increase
because an elastic supply,
supply and the entire rent is
function would not be deemed to be a surplus
fixed in total. Economic (economic rent) by
rent is economists.
more likely to decline
Answer (C) is incorrect as price rises and the
because a fixed supply quantity
schedule is typical of demanded falls.
resources that receive
economic Answer (D) is correct.
rent. When government
imposes
Answer (D) is incorrect health and safety
because market price is regulations, production
irrelevant to the cost is likely
definition. to increase as a result
of the manufacturer's
efforts to
[57] Source: CMA 0691 1- comply. The increased
19 costs are then passed on
to
Answer (A) is incorrect consumers in the form
because consumption of higher prices.
will Moreover,
either remain the amount the
unchanged or decline manufacturer is willing to
following the supply will
increase in price. decrease.

Answer (B) is incorrect


because the costs of [58] Source: CMA 0692 1-
compliance will lead to 28
higher prices.
Answer (A) is incorrect
Answer (C) is incorrect because the use of fixed
because tax revenues are
costs implies a short- be variable. Thus, the
run analysis. economist's short-run
analysis
Answer (B) is incorrect of cost is similar to the
because average product accounting concept of
is a cost in
nonsense answer. which the sum of
variable and fixed costs
Answer (C) is incorrect is total cost.
because the combination
of
two average costs [59] Source: CMA 0692 1-
produces an average 29
cost, not a
total cost. Answer (A) is incorrect
because the point of
Answer (D) is correct. diminishing average
The sum of the average productivity is the point
fixed at which
costs and the average average productivity
variable costs for a given begins to decline as
output is the average additional
total cost. Recognition of units of input are used.
fixed
costs implies a short- Answer (B) is correct.
run analysis because Marginal product is the
long-run output
analysis assumes a obtained by adding one
period long enough for all extra unit of a variable
costs to input
factor. If the cost of the
input factor is constant, a
rising marginal product [60] Source: CMA 0692 1-
will result in a declining 30
marginal cost of output.
If marginal product is Answer (A) is incorrect
falling, because an increase in
marginal cost is rising. fixed
Hence, marginal cost is costs could cause
at a average costs to
minimum when increase. Also, by
marginal product is at a definition, all long-run
maximum. costs are variable.

Answer (C) is incorrect Answer (B) is incorrect


because marginal cost is because technological
the efficiency refers to the
addition to total cost as ratio of physical output of
a result of increasing a
production by one unit. given technology and
the maximum output that
Answer (D) is incorrect is
because the point of possible. An increase in
diminishing marginal technological efficiency
productivity is that point is
at which only one of the ways
marginal productivity that economies of scale
begins to decline as can
additional occur.
inputs are added to
production. Answer (C) is incorrect
because a decline in
average
cost means the firm is
experiencing increasing [61] Source: CMA 1292 1-
returns, 1
not decreasing returns.
Answer (A) is incorrect
Answer (D) is correct. because prices might
When long-run average decline
cost or stay the same
declines as output depending upon the slope
increases, the firm is of the
experiencing demand and supply
economies of scale. curves. For example, if
Average cost falls when the
marginal demand curve were
cost is below it and steeper than the supply
rises when marginal cost curve,
is above prices would decline.
it. Average cost
reaches its minimum Answer (B) is incorrect
when it equals because prices might
marginal cost. Some of increase or stay the
the reasons for this same depending upon the
phenomenon are slope
increased specialization of the demand and
and division supply curves. For
of labor, better use and example, if the
specialization of supply curve were
management, steeper than the demand
and use of more curve,
efficient machinery and prices would increase.
equipment.
Answer (C) is correct. [62] Source: CMA 1292 1-
In a competitive market, 2
equilibrium exists when
demand is exactly equal Answer (A) is incorrect
to because the elasticity of
supply. If both demand supply is not a measure
and supply increase in involving the demand
equal curve.
amounts, the market
will still be in equilibrium, Answer (B) is correct.
but the The elasticity of supply is
new price may be a
higher, lower, or measure of the
unchanged responsiveness of a
depending upon the change in the
slopes of the demand and quantity supplied to a
supply percentage change in the
curves. Whatever the price
new price, the quantity of of the commodity. A
products cleared by the perfectly inelastic supply
market should increase. curve
has an elasticity of
Answer (D) is incorrect zero. This can occur only
because prices might when the
decline numerator of the
or stay the same elasticity fraction, the
depending upon the slope percentage
of the change in quantity
demand and supply supplied, is zero. Thus, a
curves. condition
of perfect inelasticity of [63] Source: CMA 1292 1-
supply could occur only 3
when
a firm could not vary Answer (A) is incorrect
the quantity it supplies, because the amount
i.e., demanded will increase
when the firm cannot as a result of the
vary its input usage. artificially
low price.
Answer (C) is incorrect
because a perfectly Answer (B) is incorrect
inelastic because the amount
supply curve is vertical. supplied
The firm will supply the will decline as a result
same of the artificially low
quantity at all price price.
levels.
Answer (C) is correct. A
Answer (D) is incorrect price ceiling lower than
because, in the long run, the
a equilibrium price
firm can vary its inputs. causes shortages to
Only in the short-run develop. The
situation artificially low price
is input usage results in an amount
unchangeable. Hence, supplied less
perfectly than that at the
inelastic supply is a equilibrium price. It also
short-run condition. causes
consumers to demand
more of the commodity
than at
the equilibrium price.
Answer (C) is incorrect
Answer (D) is incorrect because both variable
because shortages will and
occur. fixed costs have to be
covered. In the long run,
moreover, all costs are
[64] Source: CMA 1292 1- variable.
4
Answer (D) is correct.
Answer (A) is incorrect In long-run equilibrium,
because the equilibrium price,
price which equals marginal
is the intersection of revenue in a purely
the demand and supply competitive
curves market, will be equal to
and has nothing to do the suppliers' marginal
with what is fair to costs
consumers. of production.
At lower prices, more Allocation of the
consumers would be in economy's resources
the is optimal at this point.
market. Suppliers will continue to
benefit as long as price
Answer (B) is incorrect is greater than the
because the price would marginal
not cost; therefore,
be equal to total costs production will increase
unless the average cost up to the
were point at which marginal
the same as marginal cost exactly equals price.
cost. If
marginal cost exceeds dollar spent on product
price, suppliers would A should be the same as
have no the
incentive to produce. marginal utility for each
dollar spent on product B,
and so on. However,
[65] Source: CMA 1292 1- the principle of
5 diminishing
marginal utility must be
Answer (A) is incorrect considered. Under this
because the principle of assumption, equal
diminishing marginal increments of additional
utility states that consumption of a
marginal utility product result in less
decreases as than equal
consumption expands. additions of utility to
the consumer. For
Answer (B) is correct. A example,
rational individual when a person is
maximizes total utility thirsty, the first glass of
from income. This water tastes
objective can better than the second,
be accomplished when and the third has even
the utility obtained from less
the utility than the second.
last dollar spent on
each commodity Answer (C) is incorrect
purchased is the because the concept of
same. In other words, marginal utility is a
the marginal utility for measure associated with
each demand,
not with supply.
contribute less and less
Answer (D) is incorrect to the total product.
because marginal utility Thus, an
is not input is used up to the
associated with inputs point at which the
to the production marginal
process; it is increase in revenue
a measure associated from that input is equal
with demand. to its
marginal cost. This
means that adding
[66] Source: CMA 1292 1- variable inputs
6 to a firm's fixed inputs
will result in lower
Answer (A) is incorrect product
because consumers will costs, but only up to a
buy point. Accordingly, a firm
less of a product if will
prices are higher. increase short-run
supply as long as
Answer (B) is correct. marginal revenue
The law of diminishing (the price in a purely
returns competitive market)
states that if increasing exceeds
amounts of a variable marginal cost, whether
input are because of a lower
applied to a fixed input, marginal
there is some point cost or a higher price.
beyond
which additional units Answer (C) is incorrect
of the variable input will because costs decline up
to a
point as economies of Answer (C) is correct.
scale are achieved. As most firms expand
output,
Answer (D) is incorrect average costs of
because the supply curve production initially tend
is to decline.
independent of the The reasons for this
demand curve, which is include increased
based on specialization
consumer preferences. and division of labor,
better use and
specialization of
[67] Source: CMA 1292 1- management, and use
7 of more efficient
machinery and
Answer (A) is incorrect equipment.
because economies of Consequently, increasing
scale the size of a
refer to the savings in factory often results in
costs as production lower average costs.
increases;
less efficient labor Answer (D) is incorrect
would lead to higher because increasing
costs. factory
size will normally
Answer (B) is incorrect increase total costs, but
because output should result in
increase, although lower average costs.
average productivity may
or may
not change. [68] Source: CMA 1292 1-
8
greater. All firms should
Answer (A) is incorrect equate marginal revenue
because only goodwill (MR) and marginal cost
advertising is likely to (MC). For a competitive
be used. The monopolist firm, MR equals price at
will all output levels because
already be maximizing the
its profits because it has firm is a price taker.
no However, a monopolist
competitors. must
reduce price to raise
Answer (B) is incorrect sales, so its MR curve will
because monopolists decline with output.
produce a lower rate of Assuming no difference
output than in a between
competitive the MC curves of the
market. purely competitive firm
and the
Answer (C) is incorrect monopolist, the latter's
because the blend of downward-sloping MR
capital curve
and labor is not a will intersect the MC
consideration exclusive curve at a lower output
to the level
monopolist. than that for a
competitive firm. This
Answer (D) is correct. lower output
Competitive markets are level corresponds to a
preferable to other higher point (a higher
types of markets, such as price) on
monopolies, because the demand curve.
price is lower and output
output is less than with
[69] Source: CMA 1292 1- pure competition.
9
Answer (B) is incorrect
Answer (A) is correct. because economies and
The characteristics of diseconomies of scale
monopolistic do exist with
competition are a large monopolistic
number of firms, competition.
differentiated products,
relatively easy entry into Answer (C) is incorrect
the because advertising is an
market, some price important element in an
control by individual economy characterized
firms, and by
large amounts of monopolistic
nonprice competition. In competition.
such cases,
price exceeds marginal Answer (D) is incorrect
cost and resources are because monopolistic
under-allocated. The competition is
industry is typically characterized by
populated heterogeneous
by too many firms that products.
are too small. These
conditions are often
referred to as the waste [70] Source: CMA 1292 1-
of 10
monopolistic
competition. Prices are Answer (A) is incorrect
higher and because diminishing
returns
can exist in any market Answer (C) is incorrect
structure. because, in pure
competition,
Answer (B) is correct. the optimal output is
Pure competition is produced. Because
characterized by a resource
large number of buyers allocation is ideal, no
and sellers over- or underproduction
acting independently, occurs, and consumer
homogeneous or surplus (the difference
standardized between what
products, free entry consumers are willing to
into and exit of firms pay and what
from the they actually pay) is
market, perfect nonexistent.
information, no control
over prices, Answer (D) is incorrect
and no nonprice because all suppliers will
competition. Because charge the market
price equals price, given perfect
marginal cost, information in
allocation of resources is the economy.
optimal.
Firms produce the ideal
output, the output at [71] Source: CMA 1292 1-
which 11
average cost is lowest.
Price is lower and output Answer (A) is incorrect
greater than in any because oligopolists do
other market structure. not
attempt to decrease
demand for the product.
voluntarily restricting
Answer (B) is incorrect output.
because prices are
maintained by
restricting output. [72] Source: CMA 1292 1-
12
Answer (C) is incorrect
because increased costs Answer (A) is incorrect
do because economic cost
not necessarily result in includes not only
higher prices. explicit costs (dollars
paid), but
Answer (D) is correct. implicit costs as well;
In an oligopolistic implicit costs include
industry, a opportunity costs.
cartel can be formed to
add structure to a market Answer (B) is incorrect
with a few firms. A because opportunity
cartel arises when a costs
group of and dollar costs should
oligopolistic firms join be added, not subtracted.
together for price-fixing
purposes. This practice Answer (C) is incorrect
is illegal except in because all explicit and
international markets. implicit costs should be
Prices are fixed at an added, not subtracted.
amount
greater than would Answer (D) is correct.
occur under pure Economic cost is defined
competition. as
Members of the cartel
maintain higher prices by
the sum of all costs, [73] Source: CMA 1292 1-
both implicit and explicit, 13
of a
firm. Explicit costs Answer (A) is correct.
include direct In a competitive market
expenditures made for
to those outside the labor in which demand
firm, for example, the is stable, total wages
costs of cannot
labor, materials, and increase. If some
equipment. Implicit costs workers do increase their
are the wages,
payments that would the increases can
have been received if occur only if some
self-owned resources workers are laid
had been used outside off. Given a stable
the demand curve for labor, a
firm's business. Thus, higher
the lease payments price for this resource
forgone by will result in a decrease
not renting the firm's in the
building to others is an amount of labor
implicit demanded (a movement
cost. The return up the
necessary to keep demand curve).
resources
employed in a given Answer (B) is incorrect
enterprise (normal profit) because a maximum
is also wage
an implicit cost. that is lower than the
equilibrium wage will not
allow
workers to increase several firms; a single
their wages. seller is characteristic of
a
Answer (C) is incorrect monopoly.
because firms may
choose to Answer (C) is incorrect
become more capital because oligopolies are
intensive rather than typified by barriers to
smaller. entry; that is the reason
the
Answer (D) is incorrect industry has only a few
because supply will not firms.
decrease if the workers
are more productive after Answer (D) is correct.
gaining an increase in The oligopoly model is
wages. much
less specific than the
other market structures,
[74] Source: CMA 1292 1- but
14 there are typically few
firms in the industry.
Answer (A) is incorrect Thus, the
because oligopolies decisions of rival firms
contain do not go unnoticed.
several firms; a single Products
seller is characteristic of can be either
a differentiated or
monopoly. standardized. Prices
tend to be rigid (sticky)
Answer (B) is incorrect because of the
because oligopolies
contain
interdependence supply factors
among firms. Entry is determine price and
difficult output.
because of either
natural or created Answer (B) is incorrect
barriers. Price because both supply and
leadership is typical in demand factors
oligopolistic industries. determine price and
Under output.
price leadership, price
changes are announced Answer (C) is incorrect
first by because, in the long run,
a major firm. Once the firms can enter or exit
industry leader has an industry.
spoken,
other firms in the Answer (D) is correct.
industry match the price The distinction in
charged by microeconomics
the leader. The mutual between the short run
interdependence of the and the long
firms run is that the long run
influences both pricing is a time period long
and output decisions. enough
that all inputs,
including plant capacity,
[75] Source: CMA 1292 1- can be varied.
15 The short run is a time
period so brief that a firm
Answer (A) is incorrect has
because both demand insufficient time to vary
and the amount of all inputs.
Thus,
in the short run, the quantity of output at
quantity of one or more the constant price. The
inputs is ability to
fixed. control one's price
implies a changing price
level.
[76] Source: CMA 1292 1-
17 Answer (C) is incorrect
because a horizontal
Answer (A) is correct. A demand
firm that can control the curve implies an
price of its product is a unchanging price.
monopolist. In a pure
monopoly, the industry Answer (D) is incorrect
demand curve is also the because the supplier
monopolist's demand could
curve. Because this not sell all of its output
curve is not if the established price
perfectly elastic, the was
monopolist's demand higher than what
curve is consumers were willing
downward sloping, not to pay.
horizontal like the pure Also, a monopolist has
competitor's. no incentive to sell the
maximum that can be
Answer (B) is incorrect produced.
because a horizontal
supply
curve implies that the [77] Source: CMA 1292 1-
company will produce 18
any
Answer (A) is incorrect supply and demand, a
because a maximum surplus can never occur.
price in Pure
excess of the competition leads to
equilibrium price creates perfect equilibrium
neither a between
surplus nor a shortage. supply and demand.
Only when government
Answer (B) is incorrect intervenes with price
because a minimum price controls can a surplus or
below the equilibrium shortage occur. A
price creates neither a surplus can arise if a
surplus minimum
nor a shortage. price is set that
exceeds the equilibrium
Answer (C) is incorrect price. For
because a maximum example, if the
price minimum price is $5 and
(such as rent controls) the
set lower than the equilibrium price is $4,
equilibrium consumers will demand
price leads to fewer
shortages. Producers will goods at $5 than $4,
not be willing but producers will supply
to supply as much as more
consumers demand. goods at $5 than $4.
Thus, a surplus will
Answer (D) is correct. occur.
In the competitive model However, at a price of
of $4, supply would exactly
equal demand.
large-scale operations
[78] Source: CMA 1293 1- are required to achieve
3 low
unit costs and prices.
Answer (A) is incorrect In a natural monopoly,
because the ownership of the unit
natural resources is not cost (the long-term
a necessary factor in the average cost) of meeting
existence of a natural the
monopoly. entire demand is
minimized when the
Answer (B) is incorrect industry consists
because the ownership of of one firm. Thus,
patents is not a competition would be
necessary factor in the undesirable
existence of a because the presence
natural monopoly. of two or more firms
would
Answer (C) is correct. A prevent the realization
natural monopoly exists of the necessary
because economic and economies of
technical conditions scale.
exist in
the industry or Answer (D) is incorrect
economy that permit only because the government
one efficient is
supplier in a locale. A typically not the
natural monopoly exists supplier when a natural
when monopoly
economies of scale are exists.
very great, that is, when
very
[79] Source: CMA 1293 1- inelastic, customers
10 will continue buying the
product
Answer (A) is incorrect regardless of the price;
because reduced demand a boycott, however,
will drive the price means
downward. that consumers will
stop buying the product.
Answer (B) is correct. A
group boycott will
decrease [80] Source: CMA 1293 1-
the demand for a 28
product (shift the
demand curve to Answer (A) is incorrect
the left). This decrease because price equals
in demand should lead to marginal cost only in
a pure competition.
lower price for the
product assuming that Answer (B) is incorrect
supply is because, for a
constant (the supply monopolist,
curve does not shift). the optimal output is
less than that at which
Answer (C) is incorrect average
because supply remains cost is lowest.
unchanged in the short
run. Answer (C) is incorrect
because a monopolist's
Answer (D) is incorrect price
because, if demand is will not equal average
cost. The firm will stop
producing before [81] Source: CMA 1293 1-
reaching that level. 29

Answer (D) is correct. Answer (A) is incorrect


Whether a market is because an increase in
competitive or demand should drive
noncompetitive, a firm the price up if supply is
should produce constant.
at the level at which
marginal cost equals Answer (B) is correct.
marginal In a competitive market,
revenue. The difference prices will fall when the
between monopoly and demand curve shifts to
perfect competition is the left
reflected in the marginal or the supply curve
revenue curve. In shifts to the right.
perfect competition, the
price is a Answer (C) is incorrect
constant and therefore because a decline in the
equals marginal revenue, availability of a factor
which is represented by of production, e.g., labor,
a horizontal line. In a increases the cost of
monopoly (or in output and, if other
monopolistic factors are
competition), the price constant, decreases
declines as output the amount produced. A
increases, resulting in a decrease in supply
line of (shift of the curve to the
negative slope. left)
increases price in a
competitive market.
Answer (D) is incorrect Answer (B) is incorrect
because an increase in because the level of
the consumer income may
cost of production change a demand curve,
tends to increase price. but
not elasticity.

[82] Source: CMA 1293 1- Answer (C) is incorrect


30 because the elasticity of
supply is different from
Answer (A) is correct. the elasticity of product
Elasticity is the demand.
percentage
change in quantity Answer (D) is incorrect
demanded divided by the because the level of input
percentage change in costs affects the
price. It is a measure of elasticity of supply, not
how demand.
total revenues will be
affected given a specified
change in price. A [83] Source: CMA 1288 1-
factor affecting the price 26
elasticity
of demand is the Answer (A) is incorrect
availability of substitutes. because an increase in
As price supply will decrease
increases, buyers are the equilibrium price and
more likely to seek increase the
substitutes. equilibrium quantity
The more substitutes, exchanged.
the greater the elasticity.
Answer (B) is incorrect more goods are
because an increase in available at each price
supply but demand is
will decrease the unchanged, the price at
equilibrium price and which demand equals
increase the supply
equilibrium quantity will decline.
exchanged.

Answer (C) is incorrect [84] Source: CMA 1288 1-


because an increase in 28
supply will decrease
the equilibrium price and Answer (A) is correct. A
increase the pure monopoly consists
equilibrium quantity of
exchanged. a single firm with a
unique product. Such a
Answer (D) is correct. firm has
An increase in supply is a significant price
rightward shift in the control. For profit
upward-sloping supply maximization, it
curve. It produces until its
causes a decrease in marginal revenue equals
the equilibrium price and its marginal
thus an cost, unless marginal
increase in the quantity revenue is less than
exchanged, assuming the average
downward-sloping variable cost, which
demand curve does not will cause the firm to
shift. If shut down.
In a pure monopoly,
price will be higher and
output [85] Source: CMA 1289 1-
lower than in perfect 7
competition.
Answer (A) is incorrect
Answer (B) is incorrect because $20 is
because the monopolist calculated
is in using total fixed costs
control of the quantity instead of total costs,
supplied. Thus, the which
supply can includes total variable
be limited to produce costs.
the profit-maximizing
price. Answer (B) is incorrect
because $30 is
Answer (C) is incorrect calculated
because a monopolist using total variable
will costs instead of total
increase supply as long costs, which
as the demand curve is includes total fixed
inelastic. Inelasticity costs.
means that an increase
in price Answer (C) is correct.
will cause a less-than- The average total cost
proportionate decline in per unit
demand. is calculated by
dividing total costs (fixed
Answer (D) is incorrect + variable)
because there is only one by the number of units
price when a monopoly produced. Thus, $25,000
exists.
divided by 500 units Answer (C) is incorrect
produces a unit cost of because using only
$50. average
fixed costs ignores
Answer (D) is incorrect variable costs, which
because $25 is the increase in
average total with every unit
of fixed costs per unit produced.
and variable costs per
unit. Answer (D) is correct. A
firm should produce until
the marginal cost
[86] Source: CMA 1289 1- equals marginal revenue.
8 In the
short run, a firm in
Answer (A) is incorrect perfect competition will
because there would be continue
no production until
profit when selling marginal cost equals
price and total costs are selling price,
the same. which is also its
marginal revenue. The
Answer (B) is incorrect result is the
because equating selling short-run maximization
price of profits. As long as
to total variable costs selling
leaves nothing to cover price exceeds marginal
fixed cost, a firm should
costs. continue
producing. In the short
run in perfect
competition, the
market price equals production costs
marginal revenue decline with economies
because no firm of scale.
can affect price by its
production decisions. Answer (C) is incorrect
because total costs
increase
[87] Source: CMA 1289 1- with increased
9 production; only the
average cost per
Answer (A) is correct. unit declines.
When a firm experiences
economies of scale, the Answer (D) is incorrect
average unit cost of because changes in the
production decreases supply curve do not
as production increases. affect the demand curve.
This
phenomenon is
attributable to spreading [88] Source: CMA 0694 1-
fixed costs 2
over a greater number
of units of output. Both Answer (A) is incorrect
the because an indifference
short-run and long-run curve relates to
average costs are lower consumer desires, not to
because of economies willingness
of scale. to supply different
products.
Answer (B) is incorrect
because long-run unit Answer (B) is correct.
An indifference curve
depicts
all possible [89] Source: CMA 0694 1-
combinations of two 3
products that will
give equal utility or Answer (A) is incorrect
satisfaction to a because 0.20 equals the
consumer. The 10% decline in price
farther the indifference divided by the 50%
curve is from the origin, change in
the quantity demanded.
higher the level of
utility. Answer (B) is incorrect
because 10.00 assumes a
Answer (C) is incorrect 5% change in price; the
because an indifference correct decline in price is
curve relates to a 10%. It also does not
consumer's utility or calculate the change
satisfaction. over the
There is no relationship average.
to the income of the
consumer. Answer (C) is incorrect
because 0.10 is the
Answer (D) is incorrect percentage change in
because varying levels of price.
income and price
changes are not Answer (D) is correct. A
incorporated into product's price elasticity
the indifference curve; of
these are assumed to be demand is measured as
constant. the percentage change in
quantity demanded
divided by the
percentage change
in price. If elasticity is Answer (B) is incorrect
determined by because unitary
calculating elasticity
percentage changes (elasticity = 1.0) results
over the average, the in no change in revenue.
price
elasticity is Answer (C) is incorrect
because the demand
(150 - 100) curve
÷ [(150 + 100) ÷ 2] will not shift when
3.8 = price decreases; instead,
----------------------------- the
($50 - $45) equilibrium point will
÷ [($50 + $45) ÷ 2] move to a new position
on the
same curve. A price
[90] Source: CMA 0694 1- decrease on a product
6 with
elastic demand
Answer (A) is incorrect (elasticity > 1.0) results
because a price decrease in an increase
on in total revenue.
a product with inelastic
demand causes a Answer (D) is correct.
decrease in The concept of price
total revenue. A price elasticity
decrease on a product is of great practical
with concern to management
elastic demand causes accountants because a
an increase in total knowledge of elasticity
revenue. tells
the accountant or complementary
whether a price change products, in the number
will increase of
or decrease total consumers, or in the
revenue. If the demand tastes and preferences of
elasticity is consumers.
greater than one (i.e.,
the product is elastic), a Answer (B) is incorrect
price because a shift in the
decrease will cause an demand
increase in total revenue curve represents
because the demand changes attributable to
increases by a greater something
percentage than the other than price.
price decreases.
Answer (C) is correct.
The demand curve
[91] Source: CMA 0694 1- represents
13 the relationship
between the prices of a
Answer (A) is incorrect commodity
because a change in and the quantity
demand demanded at those
(increase or decrease) prices, holding
causes the demand curve other determinants of
to the quantity demanded
shift. Changes in constant. Movement
demand can occur as a along a demand curve
result of reflects a
changes in consumer change (increase or
income, in the price of decrease) in the quantity
substitute demanded.
example, when the
Answer (D) is incorrect price of one good
because a change in increases, the
demand demand for a
(increase or decrease) complementary good
causes the demand curve decreases.
to Margarine and butter,
shift. Changes in however, are substitutes
demand can occur as a and
result of their relationship is
changes in consumer direct. When the price of
income, in the price of one
substitute good increases,
or complementary demand for a substitute
products, in the number good also
of increases.
consumers, or in the
tastes and preferences of Answer (B) is incorrect
consumers. because cameras and
rolls of
film are examples of
[92] Source: CMA 0694 1- complementary products.
14 For
instance, when the
Answer (A) is correct. price of cameras
Goods or services are decreases,
complements if the people take more
price change of one has pictures and the demand
an inverse for rolls
relationship to the of film increases.
demand for the other. For
Answer (C) is incorrect unnoticed. Prices tend
because VCRs and video to be rigid (sticky)
cassettes are examples because of
of complementary the interdependence
products. among firms. Because
A decrease in the price competitors respond
of VCRs will result in only to certain price
increased demand of changes by
video cassettes. one of the firms in an
oligopolistic industry, the
Answer (D) is incorrect demand curve for an
because razors and razor oligopolist tends to be
blades are examples of kinked.
complementary products. Price decreases are
A usually matched by price
decrease in the price of decreases, but price
razors will result in increases are often not
increased followed.
demand for razor If other firms do not
blades. match a lower price, a
price
decrease by an
[93] Source: CMA 1294 1- oligopolist would capture
6 more of the
market. If other firms
Answer (A) is correct. match the price
An oligopoly consists of a decrease, less
few of the market will be
firms. Thus, the captured.
decisions of rivals do not
go
Answer (B) is incorrect much of a commodity is
because price changes supplied at a constant
will price.
have an effect on Conversely, a price
demand for an increase reduces the
oligopolist's product. quantity
demanded to zero when
Answer (C) is incorrect demand is perfectly
because an oligopolist elastic.
must
essentially match the Answer (B) is correct.
price of other firms in the The price elasticity of
industry. demand
is the percentage
Answer (D) is incorrect change in quantity
because an oligopolist demanded
cannot divided by the
shape its demand percentage change in
curve. price. If the
elasticity coefficient is
greater than one, demand
[94] Source: CMA 1294 1- is
7 elastic, whereas a
coefficient of less than
Answer (A) is incorrect one means
because demand is that demand is
perfectly inelastic. If the quantity
elastic when buyers are demanded
willing to purchase does not change at all
however when the price increases,
demand is said to be
perfectly inelastic.
advertising and
Answer (C) is incorrect emphasis on brand
because elasticity means names is
that substantial. In the long-
a change in price will run, prices exceed
cause a greater marginal
percentage cost and there is an
change in demand. underallocation of
resources.
Answer (D) is incorrect Firms produce less than
because the demand for the ideal output, and the
insulin is perfectly industry is populated by
inelastic, not merely too many firms that are
inelastic. too
small.

[95] Source: CMA 1294 1- Answer (B) is incorrect


8 because monopolistic
competition is
Answer (A) is correct. characterized by a large
Monopolistic competition number of
involves a large number sellers who produce
of firms offering differentiated products.
differentiated
products. Entry is Answer (C) is incorrect
usually relatively easy, because the market is
and some not
price control exists, but monopolistic, but
nonprice competition consumers may believe it
using to be so
given the differentiation
of products. If a person is
convinced by Answer (B) is incorrect
advertising that a certain because if the quantity
company is demanded does not
the only one with a change at all when the
specified quality of price
product, increases, demand is
monopoly exists for said to be perfectly
that person. inelastic.

Answer (D) is incorrect Answer (C) is correct.


because monopolistic The price elasticity of
competition is demand
characterized by a large is the percentage
number of change in quantity
sellers who produce demanded
differentiated products. divided by the
percentage change in
price. If the
[96] Source: CMA 1294 1- elasticity coefficient is
19 greater than one, demand
is
Answer (A) is incorrect elastic. Hence, an
because demand is elasticity of 2.0 is elastic.
perfectly
elastic when buyers are Answer (D) is incorrect
willing to purchase because the demand
however would
much of a commodity is be inelastic if the
supplied at a constant elasticity coefficient
price. were less than
one.
elasticity coefficient is
[97] Source: CMA 0695 1- greater than one, demand
15 is
elastic. If less than one,
Answer (A) is incorrect it is inelastic. Thus, if an
because luxury goods increase in price is
have accompanied by little
greater price elasticity. change in
They are not necessities. quantity demand,
demand is price inelastic.
Answer (B) is incorrect The lack
because the lack of of good substitutes is a
complements can lead reason for price
to greater elasticity. inelasticity.

Answer (C) is incorrect


because the population in [98] Source: CMA 0695 1-
the 16
market area has
nothing to do with price Answer (A) is incorrect
elasticity. because indifference
curves
Answer (D) is correct. have a negative slope.
Price elasticity of They reflect different
demand combinations of goods.
equals the percentage
change in quantity Answer (B) is incorrect
demanded because the farther the
divided by the curve
percentage change in from the origin, the
price. If the higher the total utility.
Answer (C) is incorrect [99] Source: CMA 0695 1-
because the principle of 17
diminishing marginal
utility applies. Answer (A) is incorrect
because a homogeneous
Answer (D) is correct. product is a key
An indifference curve assumption of perfect
represents all competition.
combinations of two
commodities that Answer (B) is incorrect
give equal utility to a because customer
consumer. The farther indifference regarding
the choice of seller is a key
indifference curve is assumption of perfect
from the origin, the competition.
higher the
level of utility. Answer (C) is incorrect
Indifference curves slope because small firm
downward to output
the right and are relative to the industry
convex to the origin is a key assumption of
because the perfect
utility of each competition.
additional unit of a good
diminishes. Answer (D) is correct.
Hence, indifference Perfect competition is
curves cannot intersect characterized by a
and are market structure with
negatively sloped. many buyers
and sellers acting
independently, a
homogeneous or
standardized product, number of firms and
free entry into and exit relative ease of market
from entry.
the market, perfect However, monopolistic
information, no control competition is also
over the characterized by
industry price, and the differentiated products,
absence of nonprice some
competition. Moreover, control over price, and
customers are indifferent considerable nonprice
about which firm they competition, e.g.,
buy from because price is through advertising and
the emphasis
only difference on brand awareness.
between one seller and
the next. Answer (C) is incorrect
because an oligopoly
consists of a few firms.
[100] Source: CMA 0695 Products may be
1-18 differentiated or
standardized, and entry is
Answer (A) is incorrect typically
because a monopoly difficult.
consists
of a single seller. Answer (D) is incorrect
because products are
Answer (B) is correct. standardized in pure
Like perfect competition, competition.
monopolistic
competition is
characterized by a large [101] Source: CMA 0695
1-19
changed. A shift in a
Answer (A) is incorrect demand curve may occur
because a rise in the when
price of one of the determinants
a substitute causes the changes. A shift to the
demand curve to shift to left
the represents a decline in
right. demand. A leftward shift
may
Answer (B) is incorrect be caused by an
because a rise in income unfavorable change in
causes the demand the tastes and
curve to shift to the right. preferences of
consumers, a decline in
Answer (C) is correct. consumer
The demand curve is the income (if the
relationship between commodity is a normal
the prices of a good), a
commodity decrease in the price of
(vertical axis) and the a substitute good, an
quantity demanded at the increase
various prices in the price of a
(horizontal axis), holding complementary product,
other or the
determinants of expectation of future
demand constant. A price decreases.
movement along
an existing demand Answer (D) is incorrect
curve occurs when the because a favorable
price is change
in consumers' tastes Answer (D) is correct. If
cause the demand curve demand is constant, an
to shift increase in supply
to the right. should result in a lower
equilibrium
price. The supply curve
[102] Source: CMA 0695 shifts to the right; that is,
1-20 more is supplied at
each price. Thus, the new
Answer (A) is incorrect intersection of the
because the increased supply and demand
supply curves is at a
will lead to a lower lower point on the
price. demand curve. The result
is an
Answer (B) is incorrect increase in the quantity
because an increase in demanded.
supply
increases quantity
demanded, not overall [103] Source: CMA 1295
demand. 1-17

Answer (C) is incorrect Answer (A) is incorrect


because pork is a because the 11th worker
substitute will
for beef. Hence, the cause total production
lower price of beef will to increase to 25 units,
shift an
demand for pork to the increment of five.
left, resulting in a lower
equilibrium price. Answer (B) is incorrect
because eight units is the
marginal product of [104] Source: CMA 1295
adding two workers to a 1-18
team of
ten workers. Answer (A) is correct.
The total revenue
Answer (C) is correct. produced
According to the table, by 11 workers would be
11 $1,225 (25 units x $49).
workers can produce With 12 workers
25 units, which is an producing a total of 28
increase units at
of five over the 20 units $47.50 each, the total
that 10 workers can revenue would be $1,330.
produce. This The
additional five units is $1,330 of revenue
known as the represents an increment
marginal physical of $105
product generated by the over the revenue
11th generated by 11 workers.
worker. Dividing
the $105 increment by
Answer (D) is incorrect the three incremental
because 25 units will be units (28
the - 25) produces marginal
total production by all revenue of $35 per unit.
11 workers, not the
marginal Answer (B) is incorrect
product added by the because $225 is the total
11th. increment when the
number of workers
increases
from 10 to 11.
increment when the
Answer (C) is incorrect number of workers
because $105 is the total increases
increase in revenue for from 10 to 11.
three additional units, not
the Answer (C) is correct.
marginal revenue per The marginal revenue
unit. product
represents the increase
Answer (D) is incorrect in total revenue resulting
because $47.50 is the from
new the additional
price per unit, not the production of an
marginal revenue. additional worker.
The total revenue
produced by 11 workers
[105] Source: CMA 1295 would be
1-19 $1,225 (25 units x $49).
With 12 workers
Answer (A) is incorrect producing
because $42 is the sales a total of 28 units at
price variance $47.50 each, the total
produced by multiplying revenue
the 28 units would be $1,330. The
times the $1.50 decline $1,330 of revenue
in price; it ignores the represents
revenue an increment of $105
produced by the extra over the revenue
units. generated by
11 workers.
Answer (B) is incorrect
because $225 is the total
Answer (D) is incorrect revenues. Thus,
because $47.50 is the concern about the effect
new the price
price per unit, not the increase will have on
marginal revenue. revenues relates to the
price
elasticity of demand.
[106] Source: CMA 1285
1-18 Answer (B) is incorrect
because the substitution
Answer (A) is correct. effect is implicit in the
As prices are increased, concept of elasticity. The
total fewer
revenue may increase substitutes, the less
or decrease depending elastic will be the
on the demand for a
price elasticity of good.
demand (percentage
change in Answer (C) is incorrect
quantity demanded ÷ because, in
percentage change in cost-volume-profit
price). If analysis, the nature of
demand is elastic the supply
(elasticity > 1.0), a price curve is not considered.
increase
will tend to reduce total Answer (D) is incorrect
revenues. If demand is because utility theory is
inelastic, a price also
increase will tend to implicit in the concept
raise total of elasticity. If demand is
elastic, consumers sloped. The reason is
derive less utility from that the demand curve
paying a faced by
higher price. Thus, the monopolist is also
elasticity is Joe's negatively sloped; that is,
dominant price
concern. must decrease to
increase sales. However,
a price
[107] Source: CMA 0696 cut to increase sales
1-3 applies not only to the
incremental units but
Answer (A) is incorrect also to all other units.
because the demand Each
curve in additional unit adds its
pure competition is price minus the sum of
perfectly elastic. the
reductions on
Answer (B) is incorrect preceding units to total
because a monopolist's revenue. Thus,
profit marginal revenue
is maximized when (change in total revenue)
price exceeds marginal declines as
revenue output rises, and the
and marginal cost. marginal revenue curve
will lie
Answer (C) is correct. below the demand
In a pure monopoly, the curve. If marginal
marginal revenue curve revenue equaled
is negatively the price change, the
(downwardly) marginal revenue and
demand
curves would be the additional consumption
same. of a product result in
smaller
Answer (D) is incorrect additions of utility to
because a monopolist the consumer. For
has no example, a
supply curve. Because glass of water on a hot
a monopolist equates day tastes great, the
marginal second
revenue and marginal glass less so, and the
cost, but marginal third even less.
revenue is
not price, different Answer (B) is incorrect
demand curves may because total utility will
result in increase with
different prices at the additional units of
same output level. Thus, product, but the
price increases are at a
and quantity supplied declining rate; the
do not have a unique principle applies
relationship. to marginal utility, not
total utility.

[108] Source: CMA 0696 Answer (C) is incorrect


1-4 because diminishing
marginal
Answer (A) is correct. utility results in a
The principle of downward-sloping
diminishing demand curve.
marginal utility states
that equal increments of
Answer (D) is incorrect equals the percentage
because the utility change in quantity
principle demanded
applies not to time but divided by the
to additional units of percentage change in
product. price. If the
demand coefficient is
greater than 1.0, demand
[109] Source: CMA 0696 is
1-5 elastic. If the
coefficient is less than
Answer (A) is incorrect 1.0, demand is
because an elasticity of inelastic. If the
2.0 is elasticity coefficient is
relatively elastic, not exactly 1.0,
perfectly elastic. demand has unitary
elasticity.
Answer (B) is incorrect
because an elasticity Answer (D) is incorrect
coefficient greater than because an elasticity
1.0 is elastic. coefficient greater than
1.0 is elastic.
Answer (C) is correct.
The price elasticity of
demand [110] Source: CMA 0696
measures the 1-6
responsiveness of a
change in quantity Answer (A) is incorrect
demanded to a change because demand is
in the price of a product. elastic
It when the coefficient
exceeds 1.0.
Answer (D) is incorrect
Answer (B) is correct. because the appropriate
Price increases can most technical term is
easily be passed along perfectly inelastic.
to consumers when
demand is
inelastic. If the demand [111] Source: CMA 0696
coefficient is less than 1-7
1.0, the
percentage change in Answer (A) is correct.
quantity demanded is Monopolistic competition
less than is
the percentage change characterized by a
in price. If demand does large number of firms
not offering
change at all (i.e., an differentiated products.
elasticity of zero), as may Entry into the market is
be relatively easy, firms
true of insulin, the have some price control,
demand for the product is and
perfectly substantial nonprice
inelastic. competition exists, such
as
Answer (C) is incorrect advertising.
because demand is
relatively Answer (B) is incorrect
inelastic when the because monopolistic
coefficient is greater competition is
than 0.0 but characterized by a
less than 1.0. relatively large
group of sellers.
Answer (C) is incorrect mandatory or artificial
because the market is prices. It often interferes
not with
monopolistic. There are the free operation of
many sellers. the market. A price
ceiling is a
Answer (D) is incorrect price below the
because products are not equilibrium point. The
homogeneous in result is a
monopolistic shortage because
competition; although consumer demand will
products may appear to exceed
be similar, they have supply.
differences in service,
quality, or other Answer (C) is incorrect
attributes. because a decrease in
demand (a leftward
shift in the demand
[112] Source: CMA 1296 curve) may
1-1 result when future
prices are expected to
Answer (A) is incorrect decline.
because a surplus arises However, given a price
when the price is set set below equilibrium,
above the equilibrium pressure on prices is
point. upward.
Supply will exceed
demand. Answer (D) is incorrect
because the price ceiling
Answer (B) is correct. will
Price fixing is the setting cause shortages.
of
will intersect the MC
[113] Source: CMA 1296 curve at a lower output
1-2 level
than that for a
Answer (A) is correct. competitive firm. This
Competitive markets are lower output
preferable to other level corresponds to a
types of markets, such as higher point (a higher
monopolies, because price) on
price is lower and output the demand curve.
greater. All firms should
equate marginal revenue Answer (B) is incorrect
(MR) and marginal cost because a monopolist
(MC). For a competitive produces less and
firm, MR equals price at charges a higher price
all output levels because than a pure
the competitor.
firm is a price taker.
However, a monopolist Answer (C) is incorrect
must because a monopolist
reduce price to raise produces less and
sales, so its MR curve will charges a higher price
decline with output. than a pure
Assuming no difference competitor.
between
the MC curves of the Answer (D) is incorrect
purely competitive firm because a monopolist
and the produces less and
monopolist, the latter's charges a higher price
downward-sloping MR than a pure
curve competitor.
curve. Accordingly, the
[114] Source: CMA 1296 pure monopolist must
1-3 reduce
price to increase sales,
Answer (A) is incorrect and the marginal revenue
because the demand curve will also be
curve of negatively sloped.
a pure competitor is Moreover,
perfectly elastic. Such a marginal revenue will
firm is a be less than price
price taker that can sell (average
all of its output at the revenue) because the
market price reductions
price but none of its necessary to
output at a higher price. increase sales apply to
all units that might
Answer (B) is incorrect otherwise
because profit is have been sold at a
maximized higher price. This
when marginal revenue relationship
equals marginal cost. explains why the
marginal revenue curve
Answer (C) is correct. A lies below
pure monopolist the demand curve. The
constitutes latter is a function of
the industry; that is, the price
negatively sloped and quantity demanded.
industry
demand curve is the Answer (D) is incorrect
pure monopolist's because the supply curve
demand is
unrelated to the market Answer (C) is incorrect
structure of the because the principle of
producer's comparative advantage
industry. states that world output
is
maximized when each
[115] Source: CMA 1296 output is produced by the
1-4 nation with the lower
opportunity cost.
Answer (A) is incorrect
because, under the law of Answer (D) is correct.
diminishing returns, if In a competitive market,
increasing amounts of a equilibrium exists when
variable demand is exactly equal
input are applied to a to
fixed input, there is some supply. If both demand
point and supply increase in
beyond which equal
additional units of the amounts, the market
variable input will still be in equilibrium,
will contribute less and but the
less to the total output. new price may be
higher, lower, or
Answer (B) is incorrect unchanged
because an opportunity depending upon the
cost slopes of the demand and
is the benefit forgone supply
by not choosing the next curves. Whatever the
best new price, the quantity of
use of a scarce products cleared by the
resource. market should increase.
supplier in a locale. A
[116] Source: CMA 1296 natural monopoly exists
1-19 when
economies of scale are
Answer (A) is incorrect very great, that is, when
because elasticity of very
demand large-scale operations
is unrelated to market are required to achieve
structure. low
unit costs and prices.
Answer (B) is incorrect In a natural monopoly,
because, if marginal the unit
costs are cost (the long-term
rising, multiple smaller average cost) of meeting
firms are preferable to a the
single entire demand is
producer. minimized when the
industry consists
Answer (C) is incorrect of one firm. Thus,
because consumer competition would be
demand is undesirable
unrelated to the market because the presence
structure of the supplier. of two or more firms
would
Answer (D) is correct. A prevent the realization
natural monopoly exists of the necessary
because economic and economies of
technical conditions scale.
exist in
the industry or
economy that permit only [117] Source: CMA 1296
one efficient 1-27
Answer (A) is correct. A Answer (D) is incorrect
merger is a business because a merger
combination in which between
an acquiring firm absorbs firms in different stages
a of the production process
second firm, and the is
acquiring firm remains in vertical.
business as a
combination of the two. A
horizontal [118] Source: CMA 0697
merger is the union of 1-1
two or more companies
that Answer (A) is correct. A
engage in the same or large capital outlay
similar activities; in other necessary to enter an
words, two firms in the industry is an entry
same industry. barrier.
Entry barriers are
Answer (B) is incorrect characteristic of
because a merger monopolistic
between a competition, oligopoly,
manufacturer and its and monopoly. In these
supplier or distributor is a market structures, the
vertical merger. barriers are successively
more
Answer (C) is incorrect difficult to overcome,
because a merger with the strongest
between barriers
firms in different existing in a monopoly
industries is a structure.
conglomerate merger.
Answer (B) is incorrect
because the minimum Answer (D) is incorrect
efficient scale is the because the production
lowest output at which possibility boundary is
long-term a macroeconomic
average unit cost is concept. It
minimized. A natural depicts the short-run
monopoly maximum gross domestic
occurs when only one product obtainable
firm can produce at the given full production and
MES. full
The reason is that employment of existing
economies of scale can resources (land, labor,
continue to capital, and
be obtained at output entrepreneurial ability).
levels in excess of the
total
market demand. [119] Source: CMA 0697
Accordingly, the lowest 1-2
unit costs
are achieved when one Answer (A) is incorrect
producer exists. because a natural
monopoly
Answer (C) is incorrect exists when economic
because a barrier to or technical conditions
entry permit
may be created by the only one efficient
firms already operating in supplier. It arises when
the economies
industry. An example is of scale are very great,
an ongoing advertising that is, when very large
campaign.
operations are needed because an
to achieve low unit costs oligopolistic industry
and usually has
prices. Thus, the unit substantial economies
cost of meeting demand of scale. Hence, a new
is entrant
minimized when the needs to begin as a
industry has one firm. large producer. Other
barriers,
Answer (B) is incorrect such as existing firms'
because a cartel is a control of technology or
group of raw
oligopolistic firms that materials, the need for
have joined together for substantial advertising,
price-fixing purposes. or
The practice is illegal costly licensing
except in requirements, may make
international markets. entry
difficult.
Answer (C) is correct.
An oligopoly is Answer (D) is incorrect
characterized because monopolistic
by a few firms in the competition is
industry (but more than characterized by a
one). relatively large
Prices tend to be rigid number of firms in the
because of the industry, differentiated
interdependence products, relatively
among firms. Entry is easy entry into the
difficult market, and
large amounts of
nonprice competition.
Answer (B) is incorrect
because superior
[120] Source: CMA 0697 (normal)
1-3 goods are defined as
those for which demand
Answer (A) is correct. is
The increase in prices at positively correlated
the with income.
movie theater caused
consumers to demand Answer (C) is incorrect
fewer because sales of a
movies at the theater complementary good
and more movies at the are negatively correlated
video with
store (where prices changes in the price of
were unchanged). Thus, its complement. For
cross-elasticity of example,
demand existed because sales of tennis balls
the decrease with an
percentage change in increase in
quantity demanded of tennis racquet prices.
videos
was correlated with the Answer (D) is incorrect
percentage change in the because public goods are
price of movie theater characterized by the
tickets. The correlation difficulty of excluding
was individuals
positive, so the goods from their benefits.
are substitutes. Examples are national
defense
and public parks.
Answer (A) is incorrect
[121] Source: CMA 0697 because $23.50 is the
1-4 variable cost of the
eighth unit.
Answer (A) is incorrect
because $90.02 is the Answer (B) is incorrect
total because $23.75 is the
fixed cost. variable cost of the
ninth unit.
Answer (B) is incorrect
because $168 is the total Answer (C) is correct.
variable cost. Marginal cost is the
incremental cost of
Answer (C) is correct. If producing one additional
seven units can be unit.
produced at an average Thus, the marginal cost
cost of $36.86 each, of the ninth unit is the
multiplying that amount increment over the
by seven produces the total cost for eight units.
total The total
cost of $258.02. cost for eight units at
$34.75 each is $278, and
Answer (D) is incorrect the
because $280 is the total total cost for nine units
cost if average total at $33.75 each is
cost for seven units were $303.75, so
$40.00. the total cost for nine
units is $25.75 greater
than the
[122] Source: CMA 0697 total for eight units.
1-5 This $25.75 is the
marginal cost
of the ninth unit. elasticity coefficient is
greater than one, demand
Answer (D) is incorrect is
because $33.75 is the elastic. If the
average cost per unit coefficient is less than
for nine units. one, demand is
inelastic. If elasticity is
calculated as the change
[123] Source: CMA 0697 over
1-6 the average, the
coefficient for senior
Answer (A) is incorrect citizens
because weekday indicates that demand
demand is is elastic.
elastic.
(150 - 82) ÷ [(150 +
Answer (B) is incorrect 82) ÷ 2]
because weekend ------------------------------ =
demand is 2.05
inelastic. (8 - 6) ÷ [(8 + 6) ÷
2]}
Answer (C) is correct. The coefficient for
The price elasticity of weekends indicates that
demand demand is
is the percentage inelastic.
change in quantity
demanded (223 - 221) ÷ [(223 +
divided by the 221) ÷ 2]
percentage change in -------------------------------
price. If the = .03
(20 - 15) ÷ [(20 + 15)
÷ 2]
fewer the firms in an
Answer (D) is incorrect industry, the greater the
because weekday barriers
demand is tend to be. Entry
elastic, and weekend barriers include the
demand is inelastic. existence of
substantial economies
of scale (low unit costs
[124] Source: CMA 0697 can be
1-17 achieved only by large
producers). They also
Answer (A) is incorrect include
because vertical barriers created by
integration is existing firms. For
the combination of a example, large
company with a supplier advertising
or a expenditures may be
customer. necessary to
compete. Control of
Answer (B) is incorrect raw materials or
because market technology is
concentration is the another barrier.
degree to which a few Consequently, patents
producers held by
dominate an industry. existing firms may
serve as an entry barrier
Answer (C) is correct. because
Entry barriers exist in all they prevent potential
market structures other competitors from using
than perfect competition. certain
The
technology. Patents are same or similar
rights granted by the activities.
federal
government to Answer (B) is incorrect
inventors to allow them because the purchase of
the exclusive a
use of their inventions grocery store by
for a specified period. another similar store,
whether or not
Answer (D) is incorrect in the same market, is a
because a patent is a horizontal merger, a
contract between the combination of two
government and an companies that engage in
inventor. the
There is no collusion. same or similar
activities.

[125] Source: CMA 0697 Answer (C) is incorrect


1-18 because a hot dog
producer's purchase of
Answer (A) is incorrect a soft drink manufacturer
because the purchase of is a
a conglomerate merger, a
grocery store by combination of
another similar store, companies in
whether or not unrelated industries.
in the same market, is a
horizontal merger, a Answer (D) is correct. A
combination of two vertical merger is a
companies that engage in combination of two
the companies, one of which
supplies
inputs for the other. A cost of the first unit of
brewer's purchase of a production.
glass
company is an Answer (D) is incorrect
example. The glass because $180 is the
company could average
supply glass for the cost of production for
brewer's bottles. two units.

[126] Source: Publisher [127] Source: Publisher

Answer (A) is incorrect Answer (A) is correct. A


because $100 is the fixed firm should employ inputs
cost of production. such that the marginal
products per dollar of
Answer (B) is correct. input are
Marginal cost is the equal. Because the
additional wage rate increased by
cost of producing one more than
more unit of output. the rental rate, the
Because marginal product of labor
total cost increased was
from $250 to $360, the reduced with respect to
marginal the marginal product of
cost of the second unit equipment. By using
is $110. more equipment and less
labor,
Answer (C) is incorrect the company's marginal
because $150 is the product for equipment
marginal will be
reduced and its
marginal product for Answer (A) is incorrect
labor will rise because revenue should
until the two are equal. rise
as the price falls when
Answer (B) is incorrect elasticity exceeds 1.0.
because the company
should Answer (B) is incorrect
use relatively less labor because revenue should
than before. Labor costs rise
went up at a greater as the price falls when
rate than equipment elasticity exceeds 1.0.
costs.
Answer (C) is correct.
Answer (C) is incorrect Price elasticity is the
because the company percentage change in
should quantity demanded
not decrease the divided by
amounts of both inputs if the percentage change
it wishes to in price. An elasticity of
maintain output and 2.5
profitability. means that the change
in demand will increase
Answer (D) is incorrect by
because the company 250% of any change in
has an price measured in
incentive to increase absolute
the equipment-output terms (the minus sign is
ratio. ignored). Hence, a 5%
price

[128] Source: Publisher


reduction increases imputed interest at 12%
demand by 12.5% (2.5 x of shareholders' equity is
5%). subtracted from the
accounting profit.
Answer (D) is incorrect Imputed
because the price decline interest is $60,000 for
will A, $36,000 for B, and
lead to increased $108,000 for C.
demand if elasticity is Accordingly, Company B
greater than has the
1.0. highest economic profit
($60,000 accounting
income
[129] Source: Publisher - $36,000 interest on
capital = $24,000).
Answer (A) is incorrect
because Company A has Answer (C) is incorrect
a because Company C's
negative economic economic profit is
profit ($30,000 - $60,000 $12,000 ($120,000 -
= $108,000).
-$30,000).
Answer (D) is incorrect
Answer (B) is correct. because Company B has
Economic profit is the the
excess highest economic
of revenues over profit.
economic costs,
including costs for
materials, labor, and [130] Source: Publisher
the cost of capital. Thus,
Answer (A) is incorrect
because Company A's Answer (B) is correct.
$30,000 income is less This type of problem is
than that of Company C. solved
by means of trial and
Answer (B) is incorrect error. Check each of the
because Company B's answer alternatives to
$60,000 income is less determine whether both
than that of Company C. points
represent a level of
Answer (C) is correct. demand for a given price.
Accounting income was Answer
given (B) is correct because
for each company. total demand would be 3
Company C had the at a
highest price of $5, and total
accounting income at demand would be 17 (5 +
$120,000. 9+
3) at a price of $1.
Answer (D) is incorrect
because the three Answer (C) is incorrect
companies because demand would
all have different levels be
of accounting income. 6, not 4, when the price
is $4, and demand would
be
[131] Source: Publisher 14, not 12, when the
price is $2.
Answer (A) is incorrect
because demand would Answer (D) is incorrect
be 1 because demand would
when the price is $6. be 6
when the price is $4 is calculated by
and 17 when the price is dividing the percentage
$1. change in
quantity demanded by
the percentage change in
[132] Source: Publisher price. The numerator
and denominator are
Answer (A) is incorrect computed
because 1.67 is the as the change over the
inverse average, which results in
of the elasticity. the
same percentage
Answer (B) is incorrect regardless of whether
because 1.06 is the result there is an
of increase or a decrease.
adding the 6% quantity Thus, the change in
decline to 1. quantity
of 3,000 units (51,500 -
Answer (C) is incorrect 48,500) divided by the
because the price average of 50,000
elasticity of [(51,500 + 48,500) ÷ 2]
demand is found by equals
dividing the 6% quantity 6%. Dividing the 6%
decline quantity decline by the
by the 10% price 10%
increase, not by adding price increase
them. produces an elasticity of
0.6.
Answer (D) is correct.
The price elasticity of
demand [133] Source: Publisher
Answer (A) is incorrect of 1,000 units (3,500 -
because 1/3 represents 2,500) divided by the
the average of 3,000
increase in quantity [(3,500 + 2,500) ÷ 2]
sold. produces a
quantity increase of
Answer (B) is incorrect 1/3. The $.50 price
because 3/2 is the decline
inverse of divided by the average
price elasticity. price of $1 produces a
price
Answer (C) is incorrect decline of 50%. Dividing
because 1 is based on the quantity increase by
the the
original quantity and price change (1/3 ÷ .5)
price rather than the equals a price elasticity
average of
quantity and price. 2/3.

Answer (D) is correct.


The price elasticity is [134] Source: Publisher
calculated by dividing
the percentage change in Answer (A) is incorrect
quantity by the because 3.00 is the
percentage change in inverse
price. The of the price increase
numerator and divided by the original,
denominator are not the
computed as the average, price.
change over the
average. Thus, the
change in quantity
Answer (B) is incorrect $1 divided by the
because 2.71 is the average price of $3.50
inverse results in a
of the correct price increase of
coefficient. 28.571%. Dividing
10.526% by
Answer (C) is correct. 28.571% results in an
The price elasticity is elasticity coefficient of .
calculated by dividing 3684,
the percentage change in or .37 rounded.
quantity demanded by
the percentage change in Answer (D) is incorrect
price. The numerator because 0.33 would be
and denominator are both the
computed as the percentage change in
change over the average, price if the original, not
which the
results in the same average, price were
percentage regardless of used in the denominator.
whether
there is an increase or
a decrease. Thus, the [135] Source: Publisher
change
in quantity demanded Answer (A) is incorrect
of 100 units divided by because $22 - $18 has a
the coefficient of 3.335
average quantity (.667 ÷ .2).
demanded of 950
produces a Answer (B) is incorrect
quantity decrease of because $18 - $14 has a
10.526%. The price coefficient of 2.668
increase of (.667 ÷ .25).
Answer (C) is incorrect Answer (A) is correct.
because $14 - $10 has a Opportunity cost is
coefficient of 1.2 (.4 ÷ . defined as
333). the return available
from the best alternative
Answer (D) is correct. investment. Since the
Inelasticity is a condition interest on the $10,000
in would
which the elasticity be $500, that is the
coefficient is less than opportunity cost of the
one. Thus, investment in the
the coefficient should saloon.
be calculated for each of
the Answer (B) is incorrect
ranges. As the price because the dividend
drops from $10 to $6, would
demand be the return on the
increases from 600 to new investment, not the
800. The quantity opportunity cost.
increased by
28.57% (200 ÷ 700). The Answer (C) is incorrect
price decline was 50% because the $10,000 is
($4 ÷ $8). Dividing the
28.57% by 50% produces cost of the investment,
an not the opportunity cost.
elasticity coefficient
of .571, which is less Answer (D) is incorrect
than one. because only the $500 of
return given up is the
opportunity cost.
[136] Source: Publisher
money at 8 units, as the
[137] Source: Publisher average total cost will be
less
Answer (A) is incorrect than revenue.
because the firm will
break
even at 5 units. [138] Source: Publisher

Answer (B) is correct. A Answer (A) is incorrect


firm should produce at because marginal costs
the are
level where marginal less than marginal
cost equals marginal revenue; thus, a rational
revenue. At firm will
6 units, marginal increase production.
revenue of $190 equals
the marginal Answer (B) is incorrect
cost. Since average because marginal costs
total cost is also $190, are
the firm less than marginal
will break even. revenue; thus, a rational
firm will
Answer (C) is incorrect increase production.
because the firm will lose
money at 8 units, as the Answer (C) is correct.
average total cost will be The firm will produce 9
less units
than revenue. because at that level
marginal cost equals
Answer (D) is incorrect marginal
because the firm will lose
revenue. At $290 per at the level where
unit, revenue will be marginal cost equals
$2,610. marginal
At $205 per unit, revenue. Competitors
expenses will be $1,845, are in long-run
leaving equilibrium
$765 profit. when price equals
minimum average cost.
Answer (D) is incorrect All of this
because marginal costs occurs at a price of
are $190.
higher than marginal
revenue; thus, profit Answer (C) is incorrect
would be because $200 is higher
less than with output of than
9 units. the minimum average
total cost.

[139] Source: Publisher Answer (D) is incorrect


because $230 is higher
Answer (A) is incorrect than
because $110 is lower the minimum average
than total cost.
the average total cost
at all levels of
production. [140] Source: Publisher

Answer (B) is correct. Answer (A) is correct.


The equilibrium price will At a price of $3, the
be quantity
demanded will be 10,
while the quantity Answer (A) is incorrect
supplied will because the price of a
be 30. Thus, there will new
be a surplus of 20 units. bicycle is irrelevant
since the bicycle
Answer (B) is incorrect purchased was
because, at a price floor not new.
of
$3, demand will exceed Answer (B) is incorrect
supply by 20 units, because $15 results from
resulting subtracting both the
in a surplus. $30 and $55 from the
price of a
Answer (C) is incorrect new bicycle.
because, at a price floor
of Answer (C) is incorrect
$3, demand will exceed because $45 results from
supply by 20 units, deducting the
resulting maximum price from the
in a surplus. new price
without regard to the
Answer (D) is incorrect price actually paid.
because, at a price floor
of Answer (D) is correct.
$3, demand will exceed The consumer surplus is
supply by 20 units, the
resulting excess of price over
in a surplus. the amount a consumer
is willing

[141] Source: Publisher


to pay. Subtracting the price at which quantity
$30 price from the $55 supplied and quantity
Jimbo demanded are equal. At
was willing to pay a price of $39, supply and
produces a consumer demand are equal at
surplus of 320 units.
$25.

[143] Source: Publisher


[142] Source: Publisher
Answer (A) is incorrect
Answer (A) is incorrect because quantity
because quantity demanded
demanded exceeds quantity
exceeds quantity supplied at this price.
supplied at this price.
Answer (B) is incorrect
Answer (B) is incorrect because quantity
because quantity demanded
demanded exceeds quantity
exceeds quantity supplied at this price.
supplied at this price.
Answer (C) is correct.
Answer (C) is incorrect The equilibrium price is
because quantity the
demanded price at which quantity
exceeds quantity supplied and quantity
supplied at this price. demanded are equal. At
a price of $42, supply and
Answer (D) is correct. demand are equal at
The equilibrium price is 285 units.
the
Answer (D) is incorrect amount at which
because, at $45, the quantity demanded and
quantity quantity
supplied is still greater supplied are equal. At a
than the quantity price of $45, the level
demanded. demanded will be 395,
the same as the quantity
supplied.
[144] Source: Publisher

Answer (A) is incorrect [145] Source: Publisher


because quantity
demanded Answer (A) is incorrect
exceeds quantity because perfect
supplied at this level. inelasticity
occurs when the
Answer (B) is incorrect coefficient is zero.
because quantity
demanded Answer (B) is correct. If
exceeds quantity the elasticity coefficient
supplied at this level. is
greater than one,
Answer (C) is incorrect demand is classified as
because quantity elastic.
demanded Since the percentage
exceeds quantity change in quantity
supplied at this level. demanded is
10% and the price
Answer (D) is correct. change is 5%, the
The equilibrium quantity elasticity
is the coefficient is 2.0 (10%
÷ 5%).
increased 10%, the
Answer (C) is incorrect quantity demanded would
because unitary decline
elasticity by less than 10%.
refers to a condition in Therefore, total revenues
which the coefficient is would
equal increase.
to one.
Answer (C) is incorrect
Answer (D) is incorrect because an inelastic
because an inelastic demand
condition exists when would lead to increased
the coefficient is less revenues as prices rise.
than one.
Answer (D) is incorrect
because demand, not
[146] Source: Publisher revenues, is inelastic.

Answer (A) is incorrect


because an inelastic [147] Source: Publisher
demand
would lead to increased Answer (A) is correct.
revenues as prices rise. Elasticity measures the
percentage change in
Answer (B) is correct. quantity demanded
Inelasticity refers to the compared
condition in which the to a percentage change
percentage change in in price. The numerator
quantity (quantity change) and
is less than the the denominator (price
percentage change in change)
price. If price
are computed as the failing to use average
change over the average. quantities and prices.
The
quantity change Answer (D) is incorrect
between points J and K because 2.50 results
is 50. from
Dividing 50 by the failing to use average
average of 75 produces a quantities and prices.
quantity
increase of 66.7%. The
price decline of $4 [148] Source: Publisher
divided
by the average of $8 is Answer (A) is incorrect
a price change of 50%. because, at a price of
Dividing 66.7% by 50% $40,
produces an elasticity of demand is 2,000 and
1.33, which is greater supply is 8,000.
than one. A demand
greater Answer (B) is incorrect
than one means because, at $38, there is
demand is defined as a
elastic. surplus of 1,500 units.

Answer (B) is incorrect Answer (C) is correct.


because .75 is the Equilibrium is defined as
inverse of the
the elasticity level at which supply
coefficient. and demand are equal.
Equilibrium occurs at a
Answer (C) is incorrect price of $36 when supply
because .40 results from and
demand are both 4,000.
will be 2,400 units,
Answer (D) is incorrect while supply will be 7,000
because, at a price of units.
$34, Thus, there will be a
demand exceeds supply surplus of 4,500 units.
by 1,500 units.

[150] Source: Publisher


[149] Source: Publisher
Answer (A) is incorrect
Answer (A) is incorrect because more Y would
because a shortage is not increase costs more
an than would be saved by
option shown on the the
chart. decline in X for a given
level of utility.
Answer (B) is incorrect
because a surplus of Answer (B) is incorrect
6,000 because both products
pounds will occur at a cannot be increased
price of $44. without violating budget
constraints.
Answer (C) is incorrect
because there is a Answer (C) is correct.
surplus, Utility is maximized when
not a shortage, since a
supply exceeds demand consumer's budget line
at a is tangent to the highest
price of $42. possible indifference
curve. A consumer should
Answer (D) is correct. be
At a price of $42, demand
indifferent between 2 calculate the total
units of Y (2 x 45 = 90) utility provided by each
and 3 of the
units of X (3 x 30 = 90). answer combinations
Since the 3 units of X using a trial-and-error
cost approach. 5M and 2N
less (3 x $10 = $30) produces a total utility of
than 2 units of Y (2 x $20 48
= (8 + 7 + 6 + 5 + 4 + 10 +
$40), the consumer 8). This answer is the
would want more units of highest of any of the
X and possible combinations.
fewer units of Y.
Answer (C) is incorrect
Answer (D) is incorrect because it produces a
because an increase in X total
will utility of only 45.
produce greater utility
for a given budget. Answer (D) is incorrect
because it produces a
total
[151] Source: Publisher utility of only 36.

Answer (A) is incorrect


because it produces a [152] Source: Publisher
total
utility of only 45. Answer (A) is incorrect
because the maximum
Answer (B) is correct. utility
The solutions approach is is 48.
to
Answer (B) is incorrect combination maximizes
because the maximum total utility. A
utility combination of
is 48. 6M and 3N produces a
total utility of 57 (8 + 7 +
Answer (C) is correct. 6
Without violating the + 5 + 4 + 3 + 10 + 8 + 6).
budget
constraint, utility can Answer (B) is incorrect
be maximized at 48. because it produces a
Maximum total
utility occurs when the utility of only 54.
quantity of M is 5 and N is
2. Answer (C) is incorrect
because it produces a
Answer (D) is incorrect total
because the maximum utility of only 46.
utility
is 48. Answer (D) is incorrect
because it produces a
total
[153] Source: Publisher utility of only 54.

Answer (A) is correct.


All of the answer [154] Source: Publisher
combinations are
within the income Answer (A) is incorrect
constraint. Thus, because the consumer is
the solutions approach not
is to determine which in equilibrium when she
is buying products whose
marginal utility are not lower price. It can be
the same per dollar of concluded that she
utility. should buy
less of A and more of B.
Answer (B) is incorrect
because product A is Answer (D) is incorrect
higher because a reduction in
priced per unit of purchases will reduce
utility. total utility.

Answer (C) is correct. A


rational consumer's [155] Source: Publisher
objective is to
maximize the total utility Answer (A) is correct. A
and the consumer will equalize
marginal utility on the the
last dollar of income. marginal utility for each
Since dollar available. Thus, if
products A and B sell the
for different prices, they marginal utility of X is
should half that of Y, then X
not have the same must
utility. B offers the same have a price equal to
utility of 3 half that of Y. Since Y is
at a price of $4. Thus, it $8, X
is logical for the must sell for $4.
consumer to
purchase more of B, Answer (B) is incorrect
which offers the same because X must be less
utility at a than
Y since it has a lower
marginal utility.
tapes ($18), resulting in
Answer (C) is incorrect a total utility of 51 (15 +
because X must be less 36).
than
Y since it has a lower Answer (D) is incorrect
marginal utility. because it violates the
income constraint.
Answer (D) is incorrect
because the price of X
will [157] Source: Publisher
be half, not double, that
of Y. Answer (A) is incorrect
because 47 is less than
the
[156] Source: Publisher 51 available under the
correct option.
Answer (A) is incorrect
because it violates the Answer (B) is correct.
income constraint. With an income of $54, a
consumer can purchase
Answer (B) is incorrect 3 CDs (a total of $36) and
because total utility is 2
only tapes ($18), resulting in
43. total utility of 51 (15 for
the 2
Answer (C) is correct. tapes and 36 for the 3
With an income of $54, a CDs).
consumer can purchase
3 CDs (a total of $36) and Answer (C) is incorrect
2 because 55 is available
only
if the income constraint Answer (D) is incorrect
is violated. because it violates the
income constraint.
Answer (D) is incorrect
because 61 is available
only [159] Source: Publisher
if the income constraint
is violated. Answer (A) is incorrect
because marginal cost
will be
[158] Source: Publisher less than average total
cost.
Answer (A) is incorrect
because the total utility Answer (B) is incorrect
is because marginal cost
only 49 (21 + 28). will be
equal to average
Answer (B) is incorrect variable cost.
because it violates the
income constraint. Answer (C) is correct.
Since fixed costs are
Answer (C) is correct. fixed in
Of the combinations total, they will decline
given, per unit as production
answers (B) and (D) increases. Thus,
violate the income average total cost will
constraint. A decrease
combination of 3 tapes when output is
and 3 CDs will maximize increased.
utility at 55 (19 + 36).
Answer (D) is incorrect capital and labor would
because average total reduce output.
cost
will decline since fixed Answer (D) is incorrect
costs are constant in because it would be more
total. advantageous to use
more labor and less
capital.
[160] Source: Publisher

Answer (A) is correct. [161] Source: Publisher


Since the marginal
product of Answer (A) is incorrect
labor is greater per because returns are
dollar of cost (10 ÷ $5 = 2 increasing.
units
per dollar) than the Answer (B) is incorrect
marginal product of because returns are
capital (5 ÷ increasing.
$10 = .5 unit per dollar,
the firm should use more Answer (C) is correct. If
labor and less capital). output increases at a
greater
Answer (B) is incorrect rate than inputs, the
because capital is more firm is experiencing
expensive than labor increasing
per unit of output. returns to scale, or
economies of scale.
Answer (C) is incorrect
because a reduction in Answer (D) is incorrect
both because the firm is
experiencing constant when a
economies of scale. doubling of inputs results
in less than
a doubling of output.
[162] Source: Publisher

Answer (A) is incorrect [163] Source: Publisher


because returns are
decreasing when Answer (A) is incorrect
output does not rise as because the $200,000 of
fast as implicit costs is not
inputs. deductible in computing
accounting income.
Answer (B) is correct.
When output increases at Answer (B) is correct.
a Implicit costs are
smaller percentage amounts that
than inputs, the firm is would have been
experiencing received if self-owned
decreasing returns to resources
scale, or had been used outside
diseconomies of scale. the firm's business.
Economic
Answer (C) is incorrect profit is pure profit, or
because the firm is the excess of revenue
experiencing over
diseconomies of scale. both explicit and
implicit costs. Revenues
Answer (D) is incorrect of $2
because returns are not million minus explicit
costs of $700,000 result
in
accounting income of $150 ($600 ÷ 4).
$1.3 million. That amount
is Answer (B) is incorrect
reduced by the because $200 is the
$200,000 of implicit costs average
to arrive at when 5 units are
economic profit of $1.1 produced.
million.
Answer (C) is incorrect
Answer (C) is incorrect because $250 is based on
because economic a
income cost of $1,000, which
will be less than was for 5 units.
accounting income when
there are Answer (D) is incorrect
implicit costs. because $600 is the total
cost, not the average
Answer (D) is incorrect cost.
because implicit costs
reduce
accounting income [165] Source: Publisher
rather than increase it.
Answer (A) is incorrect
because $150 represents
[164] Source: Publisher only the variable costs.

Answer (A) is correct. If Answer (B) is incorrect


total variable cost is because $250 represents
$600 only the fixed costs.
when 4 units are
produced, the average Answer (C) is correct.
would be The average total cost
includes both fixed and Answer (B) is incorrect
variable costs. Fixed because $300 is the
costs average
total $1,000, which variable cost of 6 units.
would be $250 per unit
when Answer (C) is incorrect
production is 4. Add the because $400 is the
$250 of average fixed average
cost variable cost for 7
to the $150 of average units.
variable cost ($600 ÷ 4)
to Answer (D) is correct.
arrive at a $400 The marginal cost of the
average total cost. seventh unit would be
equal to the increment in
Answer (D) is incorrect total
because $600 represents costs over what it costs
the to produce 6 units. Fixed
total of variable costs costs are not
rather than the average considered since they do
cost. not, by
definition, increase
with changes in
[166] Source: Publisher production. Total
variable costs increase
Answer (A) is incorrect from $1,800 to $2,800, an
because $100 is the increment of $1,000.
marginal The $1,000 is the
cost of the fourth unit. marginal cost
of the seventh unit.
[167] Source: Publisher $80,000.

Answer (A) is incorrect


because $30,000 [168] Source: Publisher
represents
only the fixed costs. Answer (A) is incorrect
because the average
Answer (B) is incorrect variable
because $50,000 cost is added to, not
represents subtracted from, the
only the variable costs. average
fixed cost.
Answer (C) is incorrect
because the total fixed Answer (B) is incorrect
costs because $3 is the
are $30,000, not $3. variable
cost, not the total cost.
Answer (D) is correct. A
firm's total costs consist Answer (C) is incorrect
of because $4 is only the
both variable and fixed fixed
costs. If the average cost per unit, not the
fixed total cost.
cost for 10,000 units is
$3, the total fixed costs Answer (D) is correct.
are At a production level of
$30,000. Adding the 1,000
$30,000 of fixed costs to units, the average fixed
the cost is $4 ($4,000 ÷ 1,000
$50,000 of variable units). Adding the $4 of
costs produces total average fixed cost to the
costs of $3
of average variable average fixed cost is
cost produces a total $40 ($200 ÷ 5), and the
cost of $7. average total cost is
$120 ($80 + $40).

[169] Source: Publisher


[170] Source: Publisher
Answer (A) is incorrect
because the marginal Answer (A) is correct.
cost For profit maximization, a
cannot be accurately firm
determined from the operating under pure
information competition should
given, but is most likely equate price
equal to the $80 of unit to marginal cost. Since
variable cost. price and marginal cost
are
Answer (B) is incorrect both $20, the firm
because the average should not change
total output.
cost is $120.
Answer (B) is incorrect
Answer (C) is incorrect because a firm should not
because the average decrease output when
fixed price is at least equal to
cost is $40. marginal cost.

Answer (D) is correct. If Answer (C) is incorrect


total variable cost is because there is no
$400 incentive
for 5 units, the average
variable cost is $80. The
to increase production than selling price. Profit
when marginal cost is is maximized when
equal to marginal
selling price. cost equals selling
price.
Answer (D) is incorrect
because a firm should not Answer (C) is incorrect
shut down as long as because a firm should not
price exceeds variable shut down as long as
cost; any selling price exceeds
excess of price over variable
variable cost provides a cost.
contribution toward the
coverage of fixed costs. Answer (D) is incorrect
because a firm in pure
competition should
[171] Source: Publisher continue increasing
production as
Answer (A) is incorrect long as the marginal
because a firm in pure cost is less than selling
competition should price.
increase production as
long as the
marginal cost is less [172] Source: Publisher
than selling price.
Answer (A) is incorrect
Answer (B) is correct. A because the monopolist
firm should continue is
increasing production not at the profit-
as long as marginal cost maximizing level when
is less marginal
revenue is lower than Answer (A) is incorrect
marginal cost. because the demand
curve
Answer (B) is incorrect would be perfectly
because a firm should not elastic.
shut down as long as
marginal revenue Answer (B) is incorrect
exceeds because the demand
variable cost. curve
would be perfectly
Answer (C) is incorrect elastic.
because the firm should
not Answer (C) is correct.
increase production The demand curve faced
when marginal cost is by a
greater firm operating under
than marginal revenue. perfect competition is
perfectly
Answer (D) is correct. A elastic (horizontal)
monopolist should not because the firm is a
continue producing at price taker. It
the current level when must sell at the market
marginal price. If the firm tries to
revenue is less than increase its price,
marginal cost. demand will drop to zero.
Decreasing output
will result in increased Answer (D) is incorrect
profits. because the curve would
be
perfectly elastic.
[173] Source: Publisher
[174] Source: Publisher Answer (A) is incorrect
because increasing price
Answer (A) is incorrect will
because the average result in lower demand
revenue for a normal good.
for 11 units is $7.
Answer (B) is correct. A
Answer (B) is incorrect monopolist should
because the average continue
revenue increasing production
for producing 11 units until marginal cost is
is $7. equal to
marginal revenue. Thus,
Answer (C) is correct. If decreasing price and
the revenue for 11 units increasing production
is will enhance profitability.
$77, and the revenue
for 10 units is $60, there Answer (C) is incorrect
is because decreasing price
marginal revenue of and output will reduce
$17 on the eleventh unit. profits.

Answer (D) is incorrect Answer (D) is incorrect


because total revenue is because increasing price
$77; will
marginal revenue is reduce demand.
$17.

[176] Source: Publisher


[175] Source: Publisher
Answer (A) is incorrect
because, at this level,
marginal costs are less
than marginal revenues. [177] Source: Publisher

Answer (B) is incorrect Answer (A) is incorrect


because, at this level, because, at a price of
marginal costs are less $1,000, only 1 unit will
than marginal revenues. be sold.

Answer (C) is correct. Answer (B) is incorrect


Profit on 1 unit would be because, at a price of
$500 ($1,000 of $600,
revenue minus $500 only 2 units will be sold.
cost). Profit for
2 units would be $680 Answer (C) is correct.
($1,200 - $520). For 3 Profits will be maximized
units, at a
profit would be $920 production level of 3
($1,500 - $580). This is units. To sell 3 units, the
the price
maximum profit since, would have to be set at
at production levels of 4 $500.
or
more, marginal cost Answer (D) is incorrect
exceeds marginal because, at a price of
revenue. $400,
sales will be 4 units,
Answer (D) is incorrect but profits will be lower
because profit is less than at 3
than units.
when 3 units are
produced.
[178] Source: Publisher
pay for 4 units.
Answer (A) is incorrect
because $1,600 would be
the total revenue based [179] Source: Publisher
on the price that the
fourth Answer (A) is incorrect
customer would be because it produces
willing to pay. lower
profits than would the
Answer (B) is correct. sale of 6 units.
The first customer is
willing to Answer (B) is correct. A
pay $1,000, the second monopolist will keep
customer $600, the third producing as long as
customer $500, and the marginal revenues
fourth customer $400, for exceed
a marginal costs. This
total revenue of $2,500. condition exists at 6
units, but not
Answer (C) is incorrect at 7. Thus, the optimal
because $2,800 would be output is at 6 units with a
the revenue if five unit
customers paid their price of $400. Any other
maximum level will result in lower
amount. profits.

Answer (D) is incorrect Answer (C) is incorrect


because it is based on because, at these levels,
the marginal revenues are
maximum amount that less than marginal costs;
the first customer is thus,
willing to
profits are less than at Answer (C) is incorrect
6 units. because the only cost
that
Answer (D) is incorrect might be related to
because, at these levels, marginal revenue would
marginal revenues are be
less than marginal costs; marginal cost.
thus,
profits are less than at Answer (D) is correct. If
6 units. a monopolist can
practice
price discrimination,
[180] Source: Publisher any marginal revenue will
be
Answer (A) is incorrect equal to the price
because the only cost charged to the next
that customer.
might be related to
marginal revenue would
be [181] Source: Publisher
marginal cost.
Answer (A) is incorrect
Answer (B) is incorrect because 6 units produce
because the only cost less
that profit for the price-
might be related to discriminating
marginal revenue would monopolist than
be does the sale of 8 units.
marginal cost.
Answer (B) is incorrect
because 7 units produce
less
profit for the price- is less than the
discriminating marginal cost.
monopolist than
does the sale of 8 units.
[182] Source: Publisher
Answer (C) is correct.
Even a price- Answer (A) is correct.
discriminating Without price
monopolist must discrimination,
evaluate the relationship profits would be $1,560
between ($2,400 of revenues for 6
marginal cost and units - $840 costs). With
marginal revenue. The price discrimination,
$300 of profits
marginal revenue at 8 would be $2,550
units is greater than the ($3,800 of revenues for 8
related units -
marginal cost of $230 $1,250 costs).
($1,250 - $1,020). The
$250 Answer (B) is incorrect
marginal revenue on because $1,400 is the
the ninth unit is less than difference in revenues,
the not profits.
$290 marginal cost
($1,540 - $1,250). Thus, Answer (C) is incorrect
profit is because $1,560 is the
maximized at 8 units. profit
at 6 units.
Answer (D) is incorrect
because the marginal Answer (D) is incorrect
revenue because $2,550 is the
profit
at 8 units.

[184] Source: Publisher


[183] Source: Publisher
Answer (A) is incorrect
Answer (A) is correct. because less labor
Marginal revenue product should be
is used.
marginal revenue ($8)
times the marginal Answer (B) is correct.
product (12 When marginal cost ($20)
units), or $96. exceeds marginal
revenue ($18), the firm
Answer (B) is incorrect should cut
because the marginal back the factors of
revenue production. Thus, less
product is the total for labor
12 units at $8 each. should be hired since
the last hour produces a
Answer (C) is incorrect $2 loss
because $10 is the cost ($20 - $18).
of
the additional Answer (C) is incorrect
production, not the because the additional
output. revenues of $18 do not
equate to profits since
Answer (D) is incorrect there
because $8 is the are additional costs.
marginal
revenue product for 1 Answer (D) is incorrect
unit, but 12 units were because the additional
produced.
revenues of $18 do not rate is 8%.
equate to profits since
there Answer (D) is incorrect
are additional costs. because a buyer would
not
be willing to pay above
[185] Source: Publisher $46,296 when the
interest
Answer (A) is incorrect rate is 8%.
because a buyer would
not
be willing to pay above [186] Source: Publisher
$46,296 when the
interest Answer (A) is incorrect
rate is 8%. because two workers
would
Answer (B) is correct. result in lower profits
The present value of than would hiring five
$50,000 workers.
one year in the future is
$46,296 at 8% interest. Answer (B) is incorrect
Thus, a buyer should be because three workers
willing to pay that would result in lower
amount or profits than would hiring
less. five
workers.
Answer (C) is incorrect
because a buyer would Answer (C) is incorrect
not because four workers
be willing to pay above would
$46,296 when the
interest
result in lower profits marginal cost, but this
than would hiring five is not true for the third
workers. worker.
Thus, only two workers
Answer (D) is correct. would be hired.
With five workers (at
total Answer (B) is incorrect
wages of $75), because marginal costs
production would be 44 would exceed marginal
units, and revenues.
revenues would be
$176 ($4 x 44). Profit Answer (C) is incorrect
would be because marginal costs
$101 ($176 - $75). Any would exceed marginal
lesser number of workers revenues.
would produce lower
profits. Note that the Answer (D) is incorrect
marginal because marginal costs
revenue of the fifth would exceed marginal
worker ($16) is greater revenues.
than the
$15 marginal cost.
[188] Source: Publisher

[187] Source: Publisher Answer (A) is incorrect


because two workers
Answer (A) is correct. would
At a wage rate of $35, not be as profitable as
the four workers.
marginal revenue of the
second worker exceeds Answer (B) is incorrect
the because three workers
would not be as
profitable as four Answer (B) is incorrect
workers. because marginal
revenue is
Answer (C) is correct. for 3 units, not 1.
At a wage rate of $35 and
with a price increase to Answer (C) is incorrect
$6, the marginal revenue because the marginal
of product
the fourth worker for the fourth unit of
exceeds the marginal resource is 3 units of
cost, but this output, not
is not true for the fifth 2.
worker. Thus, four
workers Answer (D) is correct.
would be hired. Since the fourth unit of
input
Answer (D) is incorrect results in a marginal
because the marginal product of 3 units, the
revenue marginal
produced by the fifth revenue product will be
worker would be less $15 (3 units x $5).
than the
marginal cost.
[190] Source: Publisher

[189] Source: Publisher Answer (A) is incorrect


because 3 units would be
Answer (A) is incorrect more profitable than 1.
because the marginal
product
is 3 units, not $3.
Answer (B) is correct. A Answer (A) is incorrect
firm will produce as long because $15 is total
as revenue
the marginal revenue for 15 units.
exceeds the marginal
cost. At Answer (B) is incorrect
long-run equilibrium, because $14.40 is total
marginal cost will equal revenue for 18 units.
marginal
revenue. At 3 units, the Answer (C) is incorrect
marginal product is 4 because the marginal
units of revenue
output, or $20. Thus, product is negative, not
marginal revenue equals positive.
marginal cost.
Answer (D) is correct.
Answer (C) is incorrect Revenue totals $15 at 15
because the marginal units
revenue but will decline to
is less than the $20 $14.40 if the price drops
marginal cost. to $.80.
Thus, the marginal
Answer (D) is incorrect revenue product is a
because the marginal negative $.60
revenue ($14.40 - $15.00).
is less than the $20
marginal cost.
[192] Source: Publisher

[191] Source: Publisher Answer (A) is incorrect


because labor is too
costly
relative to capital. equal MRPs per dollar
of input, so there is no
Answer (B) is incorrect advantage to
because a reduction in expanding one resource
both at the expense
inputs would reduce of the other.
profits.
Answer (B) is incorrect
Answer (C) is correct. because the resources
The firm should use less have
labor equal MRPs per dollar
and more capital of input, so there is no
because capital offers a advantage to
greater expanding one resource
marginal product per at the expense
dollar (45 ÷ $15 = 3 per of the other.
dollar)
than does labor (10 ÷ Answer (C) is incorrect
$5 = 2 per dollar). because a reduction in
both
Answer (D) is incorrect resources would reduce
because an increased profits.
use of
capital would increase Answer (D) is correct.
profits. Because both resources
have
equal MRPs, and they
[193] Source: Publisher are positive, the firm
should
Answer (A) is incorrect use more of each
because the resources resource.
have

Das könnte Ihnen auch gefallen